Davies Abdomen Questions

Pataasin ang iyong marka sa homework at exams ngayon gamit ang Quizwiz!

Hypervascularity associated with acute cholecystitis is best evaluated with Doppler interrogation of what artery below? a. common hepatic artery b. gastroduodenal artery c. pancreaticduodenal artery d. cystic artery e. proper hepatic artery

d. cystic artery

For correct orientation when performing a sagittal scan through the pancreas and aorta, which letter is on the side of the image toward the patient's feet? a. a b b c. c d. d e. e

d. d

Which letter represents the pancreatic body? a. a b. b c. c d. d e. e

d. d

Surgery has requested ultrasound guidance to excise an insulinoma. What transducer would be best for this application? a. 2.25 MHz curved array b. 3.5 MHz curved array c. 5.0 MHz curved array d. 7.0 MHz curved array e. 10 MHz linear array

e. 10 MHz linear array

You have been requested to perform a thyroid ultrasound on a patient with a palpable nodule in the left neck. Which of the following transduceres would provide the most optimal scan? a. 2.25 MHz curved linear array b. 3.5 MHz curved linear array c. 5.0 MHz curved linear array d. 7.5 MHz curced linear array e. 10.0 MHz linear array

e. 10.0 MHz linear array

Patient presents with acute right upper quadrant pain and decreasing hemat- ocrit. What is the possible diagnosis? A. Simple cyst B. Abcess C. Hemorrhagic cyst D. Parasitic cyst

C. Hemorrhagic cyst

What structure does the letter B represent? a. ligamentum teres b. falciform ligament c. hepatoduodenal ligament d. ligamentum venosum e. main lobar fissure

D. ligamentum venosum

What scanning plane is depicted in the following example? a. sagittal b. transverse c. parasagittal d. coronal e. axial

d. coronal

Cystic dilatation of the common bile duct is: a.klatskin cyst b. choledochal cyst c. mirizzi cyst d. cyst of oddi e. peribiliary cyst

b. choledochal cyst

You are performing an ultrasound exam on a patient with parathyroid hyperplasia. How many glands are usually involved in this condition? a. 1 b. 2 c. 3 d.4 e. 5

d.4

The most common anatomic variant of the gallbladder is: a. agenesis b. gallbladder folds c. phrygian cap d. duplication e. bilobed

b. gallbladder folds (second most common is phrygian cap)

You have been requested to perform a gallbladder ultrasound to rule out cholelithiasis. What is cholelithiasis? a. gallbladder carcinoma b. gallstones c. gallbladder polyps d. adenomyosis e. gallbladder wall thickening

b. gallstones

You are performing an ultrasound exam to rule oth the presence of parathyroid adenoma. Hyperparathyroidism is usually caused by enlargement of how many parathyroid glands? a. 1 b. 2 c. 3 d. 4 e. 5

a. 1

Which measurement below is the diagnostic cutoff for portal vein enlargement? a. 13mm b. 5 mm c. 10 cm d. 1.8 cm e. 9 mm

a. 13 mm (1.3 cm, should be measured during quiet respiration just where it crosses the IVC)

What is the transducer of choice for evaluation of acute appendicitis? a. 5.0 MHz linear array with short focus b. 5.0 MHz curved linear array with long focus c. 3.5 MHz curved linear array d. 3.5 MHz phased array e. 2.25 MHz phased array

a. 5.0 MHz linear array with short focus

A patient is referred from the ER to rule out acute cholecystitis. You think the gallbladder wall may be thickened. What is the normal diameter of the gallbladder wall? a. < 3mm b. <0.5mm c. <35mm d. > 3mm e. > 3cm

a. < 3mm

You suspect gut wall thickening in an elderly patient with abdominal pain and diarrhea. What is the normal average thickness of the gut wall? a. 1-2 mm b. 1-2 cm c. 3-5 mm d. 3-5 cm e. 6-7 mm

c. 3-5 mm

A referring physician has asked you about the accuracy of gallbladder sonography. The diagnostic accuracy of gallbladder sonography is: a. > 90% b. 50 % c. 100 % d. 75 % e. the diagnostic accuracy of gallbladder sonography cannot be determined

a. > 90%

Tenderness over the gallbladder with probe pressure is termed: a. Murphy's sign b. Morison's pouch c. Douglas's sign d. Tenderness of Trietz e. Courvoisier's gallbladder

a. Murphy's sign

For correct orientation when performing a transverse scan of the left kidney, which letter represents anterior? a. a b. b c. c d. d e. e

a. a

Which letter represents the uncinate process? a. a b. b c. c d. d e. e

a. a

You are scanning a patient with an enlarged caudate lobe and shrunken right lobe. What diffuse liver process should you suspect? a. cirrhosis b. acute hepatitis c. fatty infiltration d. candidiasis e. hepatocellular carcinoma

a. cirrhosis

You are scanning the liver and notic irregularity of the surface. A nodular liver surface is associated with which of the following abnormalities? a. cirrhosis b. acute hepatits c. fatty liver d. polycystic liver disease e. hepatomegaly

a. cirrhosis

What structure can be used to identify the posterior aspect of the head of the pancreas? a. common bile duct b. common hepatic artery c. gastroduodenal artery d. splenic vein e. left renal vein

a. common bile duct

You are performing a doppler study of the portal vein and detect that the flow direction in the main portal vein is the same as the hepatic artery. What does this indicate? a. a normal flow direction in the portal vein b. severe portal hypertension c. hepatofugal flow d. reversed hepatic artery flow e. portal vein/hepatic artery fistula

a. a normal flow direction in the portal vein

You are performing a doppler evaluation to rule out renal artery stenosis. You will compute a ratio comparing the velocity in the renal artery to what vessel? a. abdominal aorta b. superior mesenteric artery c. common hepatic artery d. celiac trunk e. inferior mesenteric artery

a. abdominal aorta

You are scanning a patient with increased pancreatic enzymes and white blood cell count. Which of the following conditions is most commonly associated with these labe findings? a. acute pancreatitis b. chronic pancreatitis c. adenocarcinoma of the pancreas d. islet cell tumor e. metastasis to the pancreas

a. acute pancreatitis

What is the most common cause of acute renal failure? a. acute tubular necrosis (ATN) b. renal vein thrombosis c. glomerulonephritis d. amyloidosis e. diabetes mellitus

a. acute tubular necrosis (ATN)

What is the most common malignant tumor of the gastrointestinal tract? a. adenocarcinoma b. mesenchymal tumors c. lymphoma d. metastatic tumors e. cystadenocarcinoma

a. adenocarcinoma (adenocarcinoma accounts for nearly 80% of all malignant tumors of the GI tract)

Ultrasound images obtained on a 48-year-old male show a come-tail or v-shaped reverberation artifact originating from the anterior wall of the gallbladder. This artifact most likely results from: a. adenomyomatosis b. gallbladder carcinoma c. side lobes d. porcelain gallbladder e. floating cholesterol stones

a. adenomyomatosis (comet-tail or v shaped reverberation associated with comet tail will be tapered and short compared to ringdown artifact that occurs with air from emphysematous)

A patient is referred from x-ray with a questionable left renal cyst. What are the sonographic criteria of a simple cyst? a. anechoic, acoustic enhancement, sharply defined smooth far wall, round or ovoid b. hyperechoic, acoustic enhancement, sharply defined smooth far wall, round or ovoid c. hypoechoic, acoustic attenuation, sharply defined smooth far wall, round or ovoid d. isoechoic, acoustic attenuation, sharply defined smooth far wall, round or ovoid shape e. anechoic, acoustic refraction, sharply defined smooth far wall, round or ovoid shape

a. anechoic, acoustic enhancement, sharply defined smooth far wall, round or ovoid

Which of the following is most consistent with the sonographic appearance of this renal mass? a. angiomyolipoma b. staghorn calculus c. lymphomatous involvement d. complex renal cyst e. transitional cell tumor

a. angiomyolipoma

Which of the following renal masses would most likely cause a speed propagation artifact? a. angiomyolipoma b. renal cell carcinoma c. renal pseudoaneurysm d. transitional cell carcinoma e. adenoma

a. angiomyolipoma

You are performing an ultrasound on a patient with acute pancreatitis to rule out the presence of a pseudocyst. Which retroperitoneal compartment is most frequently involved with pseudocyst? a. anterior pararenal space b. perirenal space c. posterior pararenal space d. retrofascial space e. b and c

a. anterior pararenal space

You have been asked to perform an ultrasound on a patient to rule out a leaking abdominal aortic aneurysm. Which retroperitoneal compartment contains the aorta? a. anterior pararenal space b. perirenal space c. posterior pararenal space d. retrofascial space e. the aorta is not retroperitoneal in locaction

a. anterior pararenal space

A patient has been referred for an ultrasound study of the parotid gland. Where is this gland located? a. between the external auditory canal and inferior edge of the mandible b. superior to the common carotid artery between the internal and external carotid arteries c. lateral to the sternocleidomastoid muscle on each side of the neck d. midline in the neck, superior to the isthmus of the thyroid e. medial to the carotid sheath, anterior to the longus coli muscle and inferior to the carotid bifurcation

a. between the external auditory canal and inferior edge of the mandible

You have identified a hydrocele during a scrotal ultrasound. Hydroceles form in what potential space? a. between the two layers of the tunica vaginalis b. between the tunica albuginea and the tunica vaginalis c. between the two layers of the tunica albuginea d. between the tunica albuginea and the testis e. none of the above

a. between the two layers of the tunica vaginalis (tunica albuginea surrounds the testes, tunica vaginalis is the sac that lines the inner walls of the scrotum, it has two layers parietal and visceral)

What is the etiology of the layered echoes seen in the posterior aspect of this gallbladder? a. biliary sludge b. tumefactive sludge c. floating cholesterol stones d. reverberation artifact e. gas due to emphysematous cholecystitis

a. biliary sludge

You are performing an ultrasound exam on a patient with crossed renal ectopia. Which of the following describes your findings? a. both kidneys are on the same side of the abdomen b. one of the kidneys is located in the pelvis c. one of the kidneys is located in the thoracic cavity d. the kidneys are fused together at the upper pole e. a small third kidney is located above one of the normal kidneys

a. both kidneys are on the same side of the abdomen

Which of the following describes the anatomic course of the gastroduodenal artery (GDA)? a. caudal course, anterior to the pancreatic head b. caudal course, posterior to the pancreatic head c. cranial course, anterior to the duodenum and medial to the pancreatic neck d. cranial course, posterior ro the pancreatic head e. lateral course, cephalic to the pancreatic head

a. caudal course, anterior to the pancreatic head

The orifice of the appendix opens into which of the following? a. cecum b. descending colon c. jejunum d. ileum e. sigmoid

a. cecum

A tumor that may be located in an intrahepatic or extrahepatic bile duct is known as: a. cholangiocarcinoma b. angiosarcoma c. angiomyolipoma d. cholesterolosis e. adenomyomatosis

a. cholangiocarcinoma

In the image below, the surface nodularity of the liver is indicative of: a. cirrhosis b. metastatic disease c. hepatitis d. fatty infiltration e. hydatid disease

a. cirrhosis

During transverse insonation of the thyroid gland, you detect two large vessels just lateral to the thyroid. WHat is the most medial vessel? a. common carotid artery b. internal jugular vein c. vertebral artery d. subclavian artery e. external jugular vein

a. common carotid artery

You are scanning a patient with portal hypertension. Enlargement of which of the following structures is diagnostic of this condition? a. coronary vein b. hepatic vein c. renal vein d. common bile duct e. ligamentum teres

a. coronary vein (aka left gastric, normally empties flow from the esophageal veins into the splenic vein. It becomes dilated with portal hypertension, flow direction may become reversed and this forms dangerous esophageal varices)

You are imaging a thickened loop of bowel in a patient with Crohn's disease and detect a hyperechoic "mass effect" adjacent to the bowel with a "thyroid-like" appearance. This finding is most consistent with which of the following? a. creeping fat b. fistula c. stricture d. appendicitis e. adneocarcinoma

a. creeping fat

During color doppler evaluation of the kidney, you observe inadequate fill of the intrarenal vasculature . Which doppler parameter will you adjust to improve sensitivity to flow? a. decrease pulse repetition frequency b. increase wall filter c. decrease packet size d. decreae color gain e. decrease color resolution setting

a. decrease pulse repetition frequency

What linear structure is seen on each side adjacent to the aorta and proximal to the vessels labeled "C" and "D"? a. diaphragmatic crura b. adrenal glands c. esophageal gastric junction d. normal para-aortic lymph nodes e. quadratus lumboru muscle

a. diaphragmatic crura

You are asked to perform an ultrasound study on a patient with suspected cholangiocarcinoma. What associated findings should you look for? a. dilatation of the biliary tree b. cholesterolosis c. portal hypertension d. hepatic artery pseudoaneurysm e. adenomyomatosis

a. dilatation of the biliary tree

The accessory pancreatic duct, which is sometimes visible sonographically, is known as: a. duct of santorini b. cystic duct c. duct of oddi d. duct of vater e. duct of wirsung

a. duct of santorini

Part of your routine protocol for pancreatic imaging is to comment on the echogenicity of the organ. What is the normal echogenicity of the normal pancreas? a. either isoechoic or hyperechoic compared to the liver b. always hypoechoic compared to the liver c. always hyperechoic compared to the liver d. always isoechoic compared to the liver e. the pancreatic echogenicity should never be compared to the liver

a. either isoechoic or hyperechoic compared to the liver

Ultrasound images you obtained on an 81-year-old man with acute rRUQ pain show gallstones and bright echoes in the gallbladder wall with ringdown artifacts. Which of the following is most likely? a. emphysematous cholecystitis b. gallbladder carcinoma c. cholangiocarcinoma d. acalculous cholecystitis e. umcomplicated cholelithiasis

a. emphysematous cholecystitis

Identify the structure labeled D: a. esophagus b. parathyroid c. longus coli muscle d. cervical spine e. common carotid artery

a. esophagus

During abdominal sonography of a patient with portal hypertension and splenomegaly, you detect a calcified ring at the splenic hilum. What should you do? a. evaluate the area with color doppler b. roll the patient into a decubitus position to see if the structure moves c. have the patient drink 24 oz of water and rescan d. have the patient return for a repeat study in 1 week for comparison e. nothing as this is not a significant finding

a. evaluate the area with color doppler (these findings are consistent with spenic artery aneurysm )

You have detected moderate splenomegaly and dilated, tortuous vessels at the splenic hilum. In light of these findings, you should also: a. evaluate the liver and portal vein b. look for signs of pancreatitis c. check for evidence of hematoma surrounding the spleen d. evaluate the periaortic area for lymphadenopathy e. check for an ipsilateral pleural effusion

a. evaluate the liver and portal vein (liver should be evaluated for cirrhosis, the portal vein should be evaluated for increased size and flow direction)

An ultrasound image obtained from the gallbladder shows an irregular mass within the lumen which demonstrates hypervascularity by color doppler imaging. Multiple stones are also seen within the gallbladder lumen. These findings are consistent with: a. gallbladder carcinoma b. adenomyomatosis c. tumefactive sludge d. emphysematous cholecystitis e. gallbladder perforation

a. gallbladder carcinoma

You have detected a cyst n a patient referred for sonographic evaluation of a wrist nodule. Which of the following is a cyst that occurs adjacent to a joint? a. ganglion cyst b. colloid cyst c. branchial d. dermoid e. hydatid

a. ganglion cyst

YOu are performing a thyroid study on a patient with an enlarged gland. Color doppler shows markedly increased vascularity of both lobes. Which of the following is most likely? a. grave's disease b. adenomatous goiter c. hashimoto's thyroiditis d. follicular adenoma e. colloid nodule

a. grave's disease

A patient has been referred to ultrasound to rule out the presence of pancreatic cancer. The cancer is most likely to be located in which part of the pancreas? a. head b. neck c. uncinate process d. body e. tail

a. head

Which part of the pancreas does the duodenum encircle? a. head b. body c. tail d. neck e. all of the above

a. head

Which part of the pancreas generally has the largest dimensions? a. head b. neck c. body d. tail e. uncinate process

a. head (head largest, neck smallest)

What is the most likely etiology of the liver mass in this image? a. hemangioma b. hepatocellular carcinoma c. focal nodular hyperplasia d. granuloma e. abscess

a. hemangioma

Which three structures compose the portal triad? a. hepatic artery, portal vein, and bile duct b. hepatic vein, hepatic artery, and bile duct c. hepatic vein, hepatic artery, and portal vein d. left portal vein, left hepatic artery, and ligamentum teres e. porta hepatis, main portal vein, and common bile duct

a. hepatic artery, portal vein, and bile duct

Which of the following is most commonly associated with invasion of the portal vein? a. hepatocellular carcinoma b. cavernous hemangioma c. liver metastases d. hepatic adenoma e. focal nodular hyperplasia

a. hepatocellular carcinoma

You detect a discrete echogenic focus without shadowing in the left kidney and suspect the presence of a renal calculus. Which of the following is most helpful in improving visualization of posterior acoustic shadowing? a. higher-frequency transducer b. lower-frequency transducer c. smaller-aperture transducer d. increased frame rate e. increased dynamic range

a. higher-frequency transducer

What is the normal appearance of the central sinus of the kidney? a. highly echogenic compared to the renal cortex b. hypoechoic compared to the renal cortex c. isoechoic compared to the renal cortex d. isoechoic to the medullary pyramids e. hypoechoic compared to the liver

a. highly echogenic compared to the renal cortex

During sonographic evaluation of the appendix, you detech an appendicolith. What is the sonographic appearance of this structure? a. hyperechoic focus with posterior shadowing b. hyperechoic focus with posterior acoustic enhancement c. hypoechoic focus with posterior acoustic enhancement d. hypoechoic focus with posterior shadowing e. punctate focus without shadowing

a. hyperechoic focus with posterior shadowing

You are performing a follow-up study on a patient with a renal transplant. Which of the following changes normally occurs in renal transplants compared to the immediate postoperative study? a. hypertrophy b. increased echogenicity c. hydronephrosis d. shrinkage e. calcified pyramids

a. hypertrophy

During gallbladder sonography, you notice echogenic foci within the gallbladder but do not detect distal acoustic shadowing. What changes below will improve the detectability of stone shadowing? a. increase transducer frequency, increase transducer focusing b. decrease transducer frequency, increase gain c. increase output power, decrease transducer frequency d. increase dynamic range, increase gain e. increase transducer focusing, decrease transducer frequency

a. increase transducer frequency, increase transducer focusing

Which of the following is NOT true regarding fatty liver? a. it is an irreversible disorder b. fatty liber may be caused by obesity c. it may be diffuse or focal d. it may show rapid change in appearance with time e. it commonly causes increased attenuation of the sound beam through liver

a. it is an irreversible disorder

What forms the common bile duct? a. junction of the cystic duct and common hepatic duct b. junction of the right and left hepatic ducts c. junction of the common hepatic duct, right and left hepatic ducts d. junction of the cystic duct and right hepatic duct e. junction of the duct of santorini and duct of wirsung

a. junction of the cystic duct and common hepatic duct

In this image of the gallbladder, the arrows are pointing to: a. junctional folds b. polyps c. adenomyomatosis d. rokitansky-aschoff sinuses e. tumefactive sludge

a. junctional folds

What lobe of the liver does the letter A represent? a. left lobe b. caudate lobe c. posterior right lobe d. anterior right lobe e. quadrate lobe

a. left lobe

You have detected a mass anterior and to the left of ligamentum venosum. This mass is located in what lobe of the liver? a. left lobe b. caudate lobe c. reidel's lobe d. right lobe e. quadrate lobe

a. left lobe

The left renal artery is normally located immediately posterior to which of the following? a. left renal vein b. portal vein c. common hepatic artery d. splenic artery e. none of the above

a. left renal vein

The left testicular vein drains into what vein? a. left renal vein b. inferior vena cava c. epigastric vein d. internal iliac vein e. external iliac vein

a. left renal vein

You are asked to rule out the presence of a recannalized paraumbilical. Which anatomic structure is a useful landmark in location of this structure? a. ligamentum teres b. ligamentum venosum c. coronary ligament d. hepatoduodenal ligament e. glisson's ligament

a. ligamentum teres (LPV is in contact with the ligamentum teres, a paraumbilical vein begins at the LPV and exits the liver at the ligamentum teres)

During sonographic evaluation of a 2-week-old renal transplant, you detect a fluid collection with septation and internal debris adjacent to the kidney. This most likely represents: a. lymphocele b. urinoma c. ureterocele d. hematoma e. abscess

a. lymphocele

You are performing an abdominal ultrasound on a patient with a palpable mass following kidney transplantation. Your imaging reveals an anechoic mass with multiple septations located lateral to the midline and 2 cm below the abdominal wall. Which of the following is most likely? a. lymphocele b. varices c. pancreatic pseudocyst d. hematoma e. expophytic renal cyst

a. lymphocele (lymphoceles are commonly seen following surgery)

What is the innermost lining of the gut? a. mucosa b. submucosa c. muscularis propria d. serosa e. adventitia

a. mucosa

You are performing a follow-up study on a patient diagnosed with emphysematous pyelonephritis. Which of the following best describes the sonographic appearance of this condition? a. multiple echogenic foci within the renal sinus or parenchyma with "dirty" posterior acoustic shadows b. multiple distinct hyperechoic foci with discreet, well-defined posterior acoustic shadows c. multiple indistinct, hazy foci within the renal sinus with posterior acoustic ehancement d. large area of posterior acoustic shadowing which is not associated with any defined echoes e. focal, wedge-shaped hypoechoic masses with posterior acoustic enhancement throughout the kidney

a. multiple echogenic foci within the renal sinus or parenchyma with "dirty" posterior acoustic shadows

A patient has been referred for a doppler study to rule out the presence of renal artery stenosis. A thorough knowledge of the renal vasculature is required to perform this study. Which of the following is NOT true regarding the renal vasculature? a. multiple renal arteries are rare, occuring in less than 5% of individuals b. the renal arteries lie posterior to the renal veins c. the renal arteries branch off the aorta immediately below the SMA d. the right renal vein is shorter than the left renal vein e. the renal veins drain into the IVC

a. multiple renal arteries are rare, occuring in less than 5% of individual (30%)

You are scanning a patient with known bladder outlet obstruction and note thickening of the urinary bladder wall. What is the most likely etiology of the wall thickening? a. muscular hypertrophy b. endometriosis c. hematoma d. renal cell carcinoma e. oncocytoma

a. muscular hypertrophy

A patient has been referred to your ultrasound lab with a history of acute pyelonephritis. What is the most common sonographic appearance of this condition? a. normal appearance b. irregular renal surface contour c. mottles appearance of both kidneys d. focal hypoechoic masses throughout the kidney e. gas within the renal parenchyma

a. normal appearance

What is the most common cause of acute pancreatitis? a. obstruction of the pancreatic duct by biliary calculi b. pancreatic divisum c. alcohol abuse d. trauma e. crohn's disease

a. obstruction of the pancreatic duct by biliary calculi (second most common is alcohol abuse)

You have been asked to perform an ultrasound exam on a patient with recent muscular trauma. Which of the following is true regarding the normal sonographic findings in a muscle? a. on sagittal scans, oblique, parallel, echogenic fibers are seen against a hypoechoic background b. color doppler reveals no signals in a normal muscle at rest c. ultrasound is extremely useful in identifying strained muscles d. muscle fibers cannot be recognized sonographically e. all of the above statements are true

a. on sagittal scans, oblique, parallel, echogenic fibers are seen against a hypoechoic background

You have been asked to perform an abdominal ultrasound on a patient with abnormal lipase levels. All other lab work is normal.Considering this history, what is the specific area of interest? a. pancreas b. liver c. adrenal d. kidney e. aorta

a. pancreas

YOu are performing a thyroid sonogram on a patient with a nodule suspicious for malignancy. What is the most common form of thyroid cancer? a. papillary carcinoma b. follicular carcinoma c. medullary carcinoma d. lymphoma e. metastatic disease

a. papillary carcinoma

A 52-year-old male with known liver cirrhosis presents for an abdominal ultrasound. You will carefully evaluate the liver to rule out the presence of any focal mass because of which true statement below? a. patients with liver cirrhosis are at an increased risk for hepatocellular carcinoma b. patients with liver cirrhosis tend to develop multiple cysts in their liver and pancreas c. metastatic disease occurs commonly with cirrhosis d. the presence of regenerative nodules rules out cirrhosis e. all of the above are correct

a. patients with liver cirrhosis are at an increased risk for hepatocellular carcinoma

A patient is referred for ultrasound evaluation of a questionable mass in the dome of the liver seen on a CAT scan. Which method below would improve visualization in this area of the liver? a. perform a subcostal scan with the probe angled superior and the patient in deep inspiration b. perform an intercostal scan with the probe in a coronal plane and the patient in expiration c. perform a subcostal scan with the patient performing a valsalva maneuver d. roll the patient into a right lateral decubitus position and scan from a subcostal approach with the patient in expiration e. the dome of the liver cannot be seen with ultrasound

a. perform a subcostal scan with the probe angled superior and the patient in deep inspiration

You are performing a prostate sonogram to evaluate a patient with known prostate cancer. Which anatomic zone is involved with the cancer? a. peripheral zone b. central zone c. transition zone d. periurethral glandular area e. prostate cancer occurs equally between the peripheral zone and the central zone

a. peripheral zone

A nonencapsulated collection of necrotic and edematous peripancreatic tissues is termed: a. phlegmon b. pseudocyst c. pseudoaneurysm d. ascites e. cystadenoma

a. phlegmon

What pathology is present in this image obtained from the left upper quadrant? a. pleural effusion b. splenomegaly c. ascites d. splenic metastases e. splenic cysts

a. pleural effusion

During splenic sonography, you detect moderate splenomegaly and dilated, tortuous vessels at the splenic hilum. What condition do you suspect? a. portal hypertension b. histoplasmosis c. lymphoma d. splenic rupture e. splenic pseudocyst

a. portal hypertension

WHat vessel is labeled "A" in this intercostal view of the liver? a. portal vein b. right hepatic vein c. middle hepatic vein d. left hepatic vein e. right hepatic artery

a. portal vein

Which vessel below provides the greatest amount of oxygenated flow to the liver? a. portal vein b. hepatic vein c. gastroduodenal artery d. hepatic artery e. superior mesenteric artery

a. portal vein

During insonation of the pancreas, you notice a prominent vessel just posterior to the pancreatic neck. What vessel are you imaging? a. portal-splenic confluence b. inferior mesenteric vein c. left renal vein d. superior mesenteric artery e. gastroduodenal artery

a. portal-splenic confluence

What lab test is most helpful in screening patients for prostate cancer? a. prostate specific antigen b. serum alpha-fetoprotein c. serum bilirubin d. prostate cancer detection factor e. prostate temperature

a. prostate specific antigen

You have obtained this image from the mid abdomen of a patient with a history of severe sneezing and a palpable abdominal mass. What pathology is present in this image? a. rectus sheath hematoma b. aortic aneurysm c. lymphadenopathy d. bowel tumor e. abdominal hernia

a. rectus sheath hematoma

YOu are imaging the gallbladder in a transverse orientation and detect shadowing on each edge of organ. What artifact are you identifying? a. refraction b. ringdown c. comet tail d. reverberation e. partial volume

a. refraction

What significant complication following liver transplantation is NOT detectable with ultrasound? a. rejection b. malignant disease c. hepatic artery thrombosis d. portal vein thrombosis e. pseudoaneurysm

a. rejection (liver biopsies are performed to rule out rejection)

A patient is referred for gallbladder ultrasound with a history of RUQ pain and nausea. You suspect presence of a stone in the region of the gallbladder neck, but are not sure. Which of the following would be helpful in confirming the presence of a stone? a. roll the patient into a left lateral decubitus position b. have the patient perform a valsalva maneuver c. place the patient in a trendelenburg position d. increase the system dynamic range e. increase the system overall gain

a. roll the patient into a left lateral decubitus position

Which malignant tumor is most common in children aged 2-5? a. renal hamartoma b. Wilm's tumor c. renal cell carcinoma d. transitional cell carcinoma e. renal lymphoma

b. Wilm's tumor

You are scanning a gallbladder and notice posterior acoustic shadowing. You are unsure if the shadowing is due to bowel gas or gallstones. Which of the following would be felpful in making this distinction? a. roll the patient into a left lateral decubitus position b. have the patient perform a valsalva maneuver c. use a lower frequency transducer and rescan the gallbladder d. increase the system overall gain e. increase the system dynamic range

a. roll the patient into a left lateral decubitus position

Abdominal sonography of a 42-year-old male reveals a solid mass located at the upper pole of the kidney. Which of the following would be most helpful in differentiating between a renal and adrenal mass? a. scan the patient in both deep inspiration and expiration b. ecaluate the mass with color doppler c. have the patient drink 32 oz of water and rescan d. give the patient a fatty meal and rescan in 20 minutes e. perform spectral doppler resistive indices from vessels within the mass

a. scan the patient in both deep inspiration and expiration (the adrenal glands are fixed in position, so with deep inspiration the kidney moves downward but the adrenal wouldn't)

You are imaging a patient with a high liver. Subcostal images do not clearly demonstrate the liver tissue. What should you do? a. scan the patient in deep inspiration b. scan the patient in expiration c. place the patient in trendelenburg position and rescan d. have the patient drink 32 oz of water and rescan e. scan with the patient in quiet respiration

a. scan the patient in deep inspiration

The most common germ cell testicular tumor is: a. seminoma b. teratoma c. embryonal carcinoma d. choriocarcinoma e. lymphoma

a. seminoma

The most common germ cell tumor of the testis is: a. seminoma b. yolk sac tumor c. chorion carcinoma d. embryonal cell carcinoma e. adenomatoid carcinoma

a. seminoma

You are scanning a patient with known liver cirrhosis and notice a focal mass within the posterior right lobw. What laboratory test would be the most helpful in determining if this mass is hepatocellular carcinoma? a. serum alpha-fetoprotein b. alkaline phosphatase c. serum bilirubin d. serum creatinine e. lactate dehydrogenase (LD)

a. serum alpha-fetoprotein

You are performing an ultrasound on a patient with suspected renal failure. What lab work is elevated with renal failure? a. serum creatinine b. urine creatinine c. serum bilirubin d. serum lipase e. alpha-fetoprotein

a. serum creatinine (urine creatinine is decreased with renal disease)

You are performing an ultrasound study to rule out the presence of cholelithiasis. A small echogenic foci is seen in the posterior aspect of the gallbladder fundus. How can you determine if this foci represents a polyp or a stone? a. shadowing is not present with polyps but is present with stones b. unlike a stone, a polyp should move with varying patient positions c. a stone will produce a ringdown artifact and a polyp will produce a shadow d. a polyp is always located in a dependent position e. stones are always larger than polyps

a. shadowing is not present with polyps but is present with stones

Which of the following structures is NOT retroperitoneal in location? a. spleen b. kidney c. pancreas d. aorta e. psoas muscles

a. spleen

You are scanning a patient with liver cirrhosis and suspected portal hypertension. In this study, assessment of the size of which of the following is the most important? a. spleen b. common bile duct c. abdominal aorta d. right hepatic vein e. inferior vena cava

a. spleen

Which of the following arteries are branches of the celiac trunk? a. splenic, left gastric, and common hepatic b. splenic, right gastric, and proper hepatic c. superior mesenteric, right gastric, and proper hepatic d. inferior mesenteric, right gastric, and common hepatic e. left gastric, common hepatic, and proper hepatic

a. splenic, left gastric, and common hepatic

Identify the structure labeled "C" in this image: a. stomach b. duodenum c. pancreas d. gastroesophageal junction e. crus of the diaphragm

a. stomach

Which of the following correctly describes the probe placement and imaging plane you would use to demonstrate the three hepatic veins and inferior vena cava in one view? a. subcostal oblique approach with the probe angled superiorly and to the patient's right b. intercostal approach with the probe angled inferiorly and to the patient's left c. intercostal approach with the probe oriented in a coronal plane d. subcostal oblique approach with the probe angled inferiorly and to the patien't left e. sagittal subcostal approach with the probe just to the right of midline

a. subcostal obliwue approach with the probe angled superiorly and to the patient's right

The renal arteries arise from the aorta closest to the origin of which of the following arteries? a. superior mesenteric artery b. celiac trunk c. inferior mesenteric artery d. common iliac arteries e. common hepatic

a. superior mesenteric artery

What structure is labeled E in this image? a. superior mesenteric artery b. gastroduodenal artery c. common bile duct d. commone hepatic duct e. left gastric artery

a. superior mesenteric artery

You are performing an abdominal ultrasound on a patient with a history of Crohn's disease. What part of the GI tract is most commonly involved with this disorder? a. terminal ileum and colon b. colon and antrum of the stomach c. duodenum d. pyloris and lesser curve of stomach e. crohn's disease affects the entire GI tract equally

a. terminal ileum and colon

Which of the following arteries shows a low-resistance waveform: a. testicular artery b. deferential artery c. cremasteric artery d. external iliac artery e. all of the above

a. testicular artery

What is the anatomic relationship of the CBD to the pancreas? a. the cbd is posterior to the head of the pancreas b. the cbd is anterior to the head of the pancreas c. the cbd is medial to the head of the pancreas d. the cbd is superior to the head of the pancreas e. the cbd is lateral to the tail of the pancreas

a. the CBD is posterior to the head of the pancreas

What technical problem is present in this waveform obtained from the splenic artery? a. the doppler gain is set too high b. the wall filter is set too low c. the baseline is set too low d. the pulse repetition frequency is set too high e. all of the above

a. the doppler gain is set too high

You have been asked to administer cholecystokinin to a patient. Which of the following do you expect to occur if the study is normal? a. the gallbladder will contract b. the common bile duct will dilate c. the intrahepatic biliary system will dilate d. the pancreatic duct will dilate e. the hepatic veins will dilate

a. the gallbladder will contract

You have detected a moderate splenomegaly and dilated, tortuous vessels at the splenic hilum. In light of these findings ,you should also. a. evaluate the liver and portal vein b. look for signs of pancreatitis c. check for evidence of hematoma surrounding the spleen d. evaluate the periaortic area for lymphadenopathy e. chiech for an ipsilateral pleural effusion

a. the gallbladder will contract

You are performing a scrotal ultrasound on a 21-year-old male with acute scrotal pain. You do not detect flow with color doppler in the painful testis. This finding is most consistent with which of the following: a. torsion b. orchitis c. microlithiasis d. seminoma e. none of the above

a. torsion

You are scanning a 69-year-old male with hematuria. Your ultrasound findings include right-sided hydronephrosis and a mass within the urinary bladder. Which of the following tumors most commonly occurs within the urinary bladder? a. transitional cell carcinoma b. renal cell carcinoma c. renal lymphoma d. renal hamartoma e. oncocytoma

a. transitional cell carcinoma

During sonographic imaging of the thyroid, you detect several cervical lymph nodes. Which of the follwing is NOT true regarding these lymph nodes? a. ultrasound cannot detect normal lymph nodes b. visualization of inflammatory lymph nodes in the neck is common c. most normal vercial lymph nodes have an oblong or oval shape d. the greatest dimension in normal cervical lymph nodes is the longitudinal axis e. most inflammatory lymph nodes exhibit an echogenic hilum

a. ultrasound cannot detect normal lymph nodes

You have a patient who is scheduled for pancreatic surgery following ultrasound and CT evaluation. What is the surgical procedure of choice for pancreatic cancer? a. whipple procedure b. cholecystectomy and pancreatectomy c. pancreatic transplant d. choledochojejeunostomy e. Wirsung procedure

a. whipple procedure

You are requested to perform an ultrasound evaluation of the gallbladder and biliary tree on an elderly female with a small frame. Which of the following transduceres is most suited to this task? a. 2.5 MHz phased array b. 5.0 MHz curved linear array c. 3.5 MFz linear array d. 10 MHz curved linear array e. 13 MHz linear array

b. 5.0 MHz curved linear array

What is the most accurate test for acute cholecystitis? a. ultrasound b. cholescintigraphy c. endoscopic retrograde cholangiography d. oral cholecystogram e. angiography

b. Cholescintigraphy (nuc med test that diagnosis obstruction of bile ducts)

The thin capsule surrounding the liver is known as: a. Reidel's capsule b. Glisson's capsule c. Teres capsule d. Langerhan's capsule e. Wirsung's capsule

b. Glisson's capsule

You are performing an abdominal ultrasound on a patient with mass in the pancreatic head. In order to be sure you are imaging the entire pancreatic head, you must identify the vessel that lies immediately posterior to the pancreatic head. What is this vessel? a. abdominal aorta b. IVC c. SMV d. splenic vein e. portal vein

b. IVC

You are having difficulty identifying the renal arteries in a patient referred for questionable renal artery stenosis. Which vessel below is most helpful as a landmark for the location of the renal arteries? a.celiac trunk b. SMA c. splenic vein d. IMA e. common hepatic artery

b. SMA

You are imaging a patient with an anatomic variant called "replaced" hepatic artery. In this anatomic variant, the right hepatic artery originates from what artery? a. celiac trunk b. SMA c. abdominal aorta d. IMA e. left hepatic artery

b. SMA

A patient is referred to rule out the presence of portal vein thrombus. You must evaluate the portal vein and other pertinent veins. What vessels form the portal vein? a. IMV and hepatic vein b. SMV and splenic vein c. SMV and IMV d. hepatic vein and hepatic artery e. left renal vein and splenic vein

b. SMV and splenic vein

Which vessel is located at the superior border of the pancreas? a. SMA b. celiac trunk c. left renal vein d. right renal artery e. IMA

b. celiac trunk

You are performing an ultrasound on a patient with suspected aortic aneurysm. Which of the following defines an aortic aneurysm? a. when the length of the dilated segment exceeds 3 cm b. a dilatation of the aorta exceeding 3 cm in any dimension c. a focal dilation of any segment exceeding 4 mm d. when the aorta becomes tortuous and dilated with an AP diameter greater than 4 cm e. when the diameter of the aorta is greater than 2.5 cm

b. a dilatation of the aorta exceeding 3 cm in any dimension

The testicular artery is a branch of what artery? a. deferential b. abdominal aorta c. internal iliac d. external iliac e. left renal

b. abdominal aorta

You are performing a transrectal ultrasound in a patient with acute prostatitis. An anechoic mass is seen within the prostate. This most likely represents: a. prostate utricle cyst b. abscess c. hematoma d. lymphocele e. seroma

b. abscess ( seen in patients with acute prostatitis that is resistant to treatment)

A patient is referred for abdominal ultrasound with a high fever and RUQ pain. You document the presence of a large, rounded, homogenous mass with low-level internal echoes and poorly defined borders. The mass is located in the right lobe of the liver, adjacent to the capsule and shows increased through transmission. This likely represents: a. hemorrhagic cyst b. abscess c. hematoma d. choledochal cyst e. loculated ascites

b. abscess (hemorrhagic cyst would have a similar appearance, but the wall would be clearly defined rather than poorly distinguished)

Which of the following is NOT a sonographic sign of tendinitis? a. thickening of the tendon b. absence of vasculary on color doppler imaging c. decreased echogenicity d. blurred margins e. calcifications in chronic tendinitis

b. absence of vasculary on color doppler imaging

During routine sonographic evaluation of the spleen, you detect a small, rounded mass at the splenic hilum that is homogeneous and isoechoic with the spleen. You have most likely detected: a. exophytic splenic cyst b. accessory spleen c. enlarged lymph node d. splenic metastasis e. splenic hemangioma

b. accessory spleen

You are scanning a patient with a history of liver transplantation. You should search for all of the following complications of this surgery EXCEPT: a. biliary sludge b. acute cholecystitis c. portal vein stenosis d. hepatic artery thrombosis e. liver malignancy

b. acute cholecystitis ( the donor gallbladder is excised during transplant surgery, don't waste time searching for a gallbladder)

Rokitansky-Aschoff sinuses are associated with which of the following conditions? a. gallbladder carcinoma b. adenomyomatosis c. tumefactive sludge d. emphysematous cholecystitis e. gallbladder perforation

b. adenomyomatosis (they are small mucosal herniations into the muscular layer of the gallbladder. Appear as cystic spaces, if filled with bile or cholesterol deposits they will appear as echogenic foci with v shaped comet tail)

Which of the following is the biggest risk factor for rupture of an AAA? a. aneurysm location b. aneurysm size c. patient age d. flow velocity through the aneurysm e. presence of laminar thrombus within the aneurysm

b. aneurysm size

You are performing an ultrasound on a patient with known horseshoe kidneys. Where is the isthmus of a horseshoe kidney located? a. in the iliac fossa b. anterior to the abdominal aorta c. pouch of douglas d. morison's pouch e. posterior to the abdominal aorta

b. anterior to the abdominal aorta

What vessel is seen posterior to the vessel labeled D? a. IVC b. aorta c. right renal artery d. IMV e. splenic vein

b. aorta

You are scanning a patient with Marfan's syndrome. Ultrasound findings reveal a linear band throughout the length of the abdominal aorta that is mobile in relation to the cardiac cycle. What pathology is most likely present? a. aortic pseudoaneurysm b. aortic dissection c. micotic aortic aneurysm d. inflammatory aortic aneurysm e. fibromuscular dysplasia of the abdominal aorta

b. aortic dissection

FOr correct orientation when performing a sagittal scan of the liver, which letter represents posterior? a. a b. b c. c d. d e. posterior is not represented on a sagittal scan

b. b

Which of the following shows the correct plane for measuring the diameter of this aortic aneurysm? a. a b. b c. c d. d e. both b and c are correct

b. b

Which of the following describes the normal course of the left renal vein? a. retroaortic b. between the superior mesenteric artery and the aorta c. anterior to the superior mesenteric and inferior vena cava d. posterior to the inferior vena cava e. between the superior mesenteric artery and the splenic vein

b. between the superior mesenteric artery and the aorta

You are performing an ultrasound exam on a patient who has been on dialysis for 4 years. Which of the following describes the typical appearance of the kidney in these cases? a. bilateral renal enlargement with increased echogenicity b. bilaterally small, echogenic kidneys with multiple cysts of varying sizes c. bilateral small, hypoechoic kidneys with increased corticomedullary distinction d. normal appearing kidneys bilaterally e. all of the above sonographic appearances are common in this scenario

b. bilaterally small, echogenic kidneys with multiple cysts of varying sizes

You are evaluating a suspicious lesion to look for gas bubbles to confirm the presenct of liver abscess in a patient with fever and increased white blood cell count. WHat is the sonographic appearance of the gas bubbles? a. brightly echogenic echoes with clean distal acoustic shadow b. brightly echogenic foci associated with echogenic ringdown artifact c. hypoechoic areas within the mass associated with increased through transmission d. anechoic foci with distal acoustic enhancement e. hyperechoic foci with distal acoustic enhancement

b. brightly echogenic foci associated with echogenic ringdown artifact

You are examining the IVC to rule out obstruction. Within the liver, the IVC lies along the posterior surface of what lobe? a. left lobe b. caudate lobe c. anterior segment right lobe d. posterior segment right lobe e. quadrate lobe

b. caudate lobe

You are imaging the right lower quadrant in a patient with leukocytosis and a low-grade fever. WHat part of the GI tract will you most likely see in this area? a. sigmoid b. cecum c. descending colon d. duodenum e. pylorus

b. cecum

You are scanning a 52-year-old male with a history of alcohol abuse. Ultrasound findings include a hyperechoic mass in the head of the pancreas, dilatation of the pancreatic and common bile duct, and diffuse calcification within the pancreas. Which of the following conditions is most likelt present? a. acute pancreatitis b. chronic pancreatitis c. adenocarcinoma d. cystadenocarcinoma e. islet cell tumor

b. chronic pancreatitis

You are scanning a patient with a history of alcohol abuse and liver cirrhosis. The pancreatic tissue is heterogeneous. Calcifications and dilatation of the pancreatic duct is present. Which condition is most likely considering this history and findings? a. acute pancreatitis b. chronic pancreatitis c. adenocarcinoma of the pancreas d. islet cell tumor e. metastasis to the pancreas

b. chronic pancreatitis

A referring physician has asked if your department uses color doppler during transrectal prostate sonography. What is the role of color doppler in this procedure? a. color doppler has not been shown to be helpful during prostate sonography b. color doppler allows improved perception of pathologic vessels associated with cancer c. increased color doppler signals are diagnostic of chronic prostatitis d. absence of flow with color doppler is indicative of benign prostatic hypertrophy e increased color doppler signals in the seminal vesicles indicates the presence of seminal vesicle cysts

b. color doppler allows improved perception of pathologic vessels associated with cancer

The transverse diameter measurements of the gallbladder in a fasting patient measure 5.3 cm. This measurement is : a. within normal limits b. consistent with a hydropic gallbladder c. consistent with an abnormally contracted gallbladder d. diagnostic of chronic cholecystitis e. diagnostic of a phrygian cap deformity of the gallbladder

b. consistent with a hydropic gallbladder

The patient you are scanning has eaten breakfast prior to your study. What is the appearance of the gallbladder in the postprandial state? a. dilatation of a thin-walled gallbladder b. contraction of the gallbladder with diffuse wall thickening c. nonvisibility of the gallbladder due to complete contraction d. contraction of the gallbladder with asymmetric wall thickening e. minimal contraction of the gallbladder with a sludge filled lumen

b. contraction of the gallbladder with diffuse wall thickening

You are imaging a 25-year-old female with ahistory of chronic diarrhea, weight loss, abdominal pain, and fever. You detect a segment of concentrically thickened small bowel that is hyperemic by color doppler. This finding is most consistent with which of the following? a. adenocarcinoma b. crohn's disease c. acute appendicitis d. acute diverticulitis e. lymphoma

b. crohn's disease

What is the purpose of scanning the urinary bladder to identify ureteral "jets"? a. rule out the presence of urinoma b. determine if a ureter is obstructed c. determine if bladder outlet obstruction is present d. search for bladder carcinoma e. identify the urethra

b. determine if a ureter is obstructed

During a routine abdominal sonogram, you detect several small lymph nodeas adjacent to the abdominal aorta. Your exam should include which of the following? a. spectral doppler analysis of each lymph node b. diameter measurements of each lymph node c. graded compression of each lymph node d. evaluation of the pelvis e. examination with the patient in a prone position

b. diameter measurements of each lymph node

You are imaging a patient with a fusiform abdominal aortic aneurysm. Which of the following describes the fusiform aneurysmal shape? a. bulbous enlargement with a sharp junction between the normal and abnormal segment b. dilatation with a gradual transition between the normal and abnormal segment c. figure eight appearance to the aneurysm d. intimal flap within the aneurysm e. sharp, sudden sac-like protuberance

b. dilatation with a gradual transition between the normal and abnormal segment

You have obtained this image from the RUQ of a patient with abdominal pain, nausea, and vomiting. What pathology is present? a. gallstones b. dilated common bile duct c. portal vein thrombosis d. pneumobilia e. pancreatic pseudocyst

b. dilated common bile duct

You have been asked to perform an ultrasound to rule out the presence of Budd-Chiari syndrome. You will tailor your exam to include which of the following? a. volume measurement of the spleen b. doppler analysis of the hepatic venous system c. both supine and upright views of the porta hepatis d. oblique view of the right lobe of the liver to include the right hemidiaphragm e. careful search for periaortic lymphadenopathy

b. doppler analysis of the hepatic venous system

Which part of the GI tract curves around the pancreatic head? a. hepatic flexure of the colon b. duodenum c. jejunum d. pyloris e. ileum

b. duodenum

You have obtained this image while performing a scrotal ultrasound on a patient with a history of a palpable right sctrotal mass. What pathology is present? a. seminoma b. epididymal cyst c. epididymitis d. orchitis e. hydrocele

b. epididymal cyst

You are scanning a patient post biopsy and discover a cystic mass in the kidney. What should you do? a. nothing, cystic masses are very common b. evaluate the cyst with color doppler c. have the patient return in 2 weeks for a follow-up study d. scan the patient in a prone position e. compress the mass with probe pressure

b. evaluate the cyst with color doppler (following biopsy, new cystic mass could be a pseudoaneurysm color will confirm)

Considering the image below, which of the following would be most likely to confirm the findings? a. roll the patient into a right lateral decubitus postion b. evaluate the hypoechoic areas with color and spectral doppler c. have the patient drink approximately 48 oz of water and rescan d. have the patient perform a valsalva maneuver e. scan the patient during both inspiration and expiration

b. evaluate the hypoechoic areas with color and spectral doppler

You are performing a doppler study of a renal transplant to rule out stenosis of the renal artery. The renal artery is usually anastomosed to which artery? a. internal iliac artery b. external iliac artery c. common iliac artery d. abdominal aorta e. inferior mesenteric artery

b. external iliac artery

You suspect enlargement of the caudate lobe in a patient with liver disease. What structure located at the anterior border of the caudate lobe will help you to identify this lobe of the liver? a. left portal vein b. fissure for the ligamentum venosum c. inferior vena cava d. fissure for the ligamentum teres e. main lobar fissure

b. fissure for the ligamentum venosum

During abdominal sonography, you detect a fluid filled mass in the left upper quadrant. What technique below is most useful in determining if this mass is the stomach or a pathologic structure? a. have the patient perform a valalva maneuver b. have the patient drink some water while imaging the suspicious area c. scan the patient in both inspiration and expiration d. roll the patient into a left lateral decubitus position and rescan e. evaluate the area with color doppler

b. have the patient drink some water while imaging the suspicious area

You have been asked to identify the lobes of the liver in a patient with a hepatic mass. Which vessels course intersegmental in the liver? a. portal veins b. hepatic veins c. hepatic arteries d. both the portal veins and hepatic veins e. both the portal veins and hepatic arteries

b. hepatic veins

Which term can be used to describethe normal sonographic appearance of the splenic parenchyma? a. heterogeneous with mid-to low-level echogenecity b. homogeneous with mid- to low- level echogenicity c. homogeneous with echogenicity exceeding that of the liver d. heterogeneous with mixed echogenicity e. the normal spleen has a varied appearance on ultrasound

b. homogeneous with mid- to low- level echogenicity

Which of the following is associated with infestation by a parasite and is most prevalent in sheep and cattle-raising countries? a. budd-chiari syndrome b. hydatid disease c. candidiasis d. hepatitis A e. Kaposi's sarcoma

b. hydatid disease

What is the most common ultrasound appearance of pancreatic adenocarcinoma? a. hyperechoic mass b. hypoechoic mass c. cystic mass d. calcified mass e. complex mass with both cystic and solid components

b. hypoechoic mass

Which of the following describes the most common appearance of seminal vesicles by transrectal prostate sonography? a. hyperechoic, symmetrical, irregularly shaped structures b. hypoechoic, symmetrical, irregularly shaped structures c. hyperechoic, asymmetrical, smooth structures d. hyperechoic, asymmetrical, irregularly shaped structures e. hypoechoic, asymmetrical, smooth structures

b. hypoechoic, symmetrical, irregularly shaped structures

You are assisting a physician in a thyroid biopsy. Which of the following would result in the best image of the needle? a. image the needle at near zero degrees incidence b. image the needle at near perpendicular incidence c. decrease the overall gain so that the needle is the only bright structure seen d. use a curved array rather tha a linear array transducer e. all of the above

b. image the needle at near perpendicular incidence

You are performing an ultrasound exam on a patient with a history of repeated bouts of pancreatitis. What would you most likely use color doppler for in this study? a. evaluate for increased flow in the pancreatic parenchyma b. improve detectability of possible pseudoaneurysms c. look for flow direction in the SMA d. assess the quality of flow in the abdominal aorta e. rule out thrombus in the IVC

b. improve detectability of possible pseudoaneurysms (color confirms patency of the splenic, portal, superier mesenteric, and hepatic vessels and to improve detectability of possible pseudoaneurysms)

You are performing a doppler study of the superior mesenteric artery. The doppler waveform is aliased. What should you do to correct this problem? a. decrease the wall filter b. increase the pulse repetition frequency c. increase the doppler frequency d. decrease the doppler gain e. raise the baseline

b. increase the pulse repetition frequency

You have been asked to obtain an abdominal ultrasound on a 320-pound male. Which of the following will help you obtain adequate penetration of the liver in this patient? a. high-frequency transducer b. increase the transmit power c. increase the receiver gain d. use a small footprint transducer e. all of the above

b. increase the transmit power

You are scanning an obese patient to rule out fatty liver. Which of the following describes a common sonographic appearance of this condition? a. increased through transmission throughout the hypoechoic liver b. increased echogenicity of the liver compared to the normal kidney c. focal hypoechoic masses throughout both lobes of the liver surrounded by normal liver echotexture d. shrunken liver with surface nodularity e. enlarged, hypoechoic right lobe compared to a small and shrunken left lobe

b. increased echogenicity of the liver compared to the normal kidney

You are performing an ultrasound study on a 27-year-old male with scrotal pain to rule out epididymo-orchitis versus torsion. Which finding is most consistent with epididymitis? a. absence of flow by color doppler b. increased flow by color doppler c. small cyst in the epididymal head d. small calcification in the epididymal head e. multiple tiny calcifications throughout the testis

b. increased flow by color doppler

You are performing an ultrasound to rule out the presence of abdominal aortic aneurysm (AAA). What is the most common location of AAA? a. suprarenal b. infrarenal c. thoracic d. aortic arch e. suprarenal and infrarenal aortic aneurysms occur with equal frequency

b. infrarenal

You are examining the spleen during routine sonography. What view provides the best long axis image of the spleen? a. sagittal view b. intercostal coronal view with patient supine c. coronal view with patient in left lateral decubitus position d. anterior sagittal view with a patient in left lateral decubitus position e. posterior sagittal view with patient in a prone position

b. intercostal coronal view with patient supine

WHich intrarenal arteries course alongside the renal pyramids? a. segmental b. interlobar c. arcuate d. interlobular e. vasa recta

b. interlobar

The prostaticovesical arteries that supply the prostate are branches from which artery? a. abdominal aorta b. internal iliac c. external iliac d. inferior mesenteric e. superior mesenteric

b. internal iliac

During transverse insonation of the thyroid gland, you detect two large vessels just lateral to the thyroid. What is the most lateral vessel? a. common carotid artery b. internal jugular vein c. vertebral artery d. subclavian artery e. external jugular vein

b. internal jugular vein

Identify the structure labeled E a. common carotid artery b. internal jugular vein c. external jugular vein d. longus coli muscle e. subclavian vein

b. internal jugular vein

What is a normal measurement for a testicle: a. 4 x 2 x 3 cm b. 2 x 3 x 4 cm c. 4 x 3 x 3 cm d. 3 x 4 x 6 cm

c. 4 x 3 x 3 cm

You have detected a dilated bowel with multiple concentric rings within it in a patient with severe abdominal pain. This finding is consistent with which of the following? a. crohn's disease b. intussesception c. adenocarcinoma d. lymphoma e. acute diverticulitis

b. intussesception

Which of the following is an endocrine tumor? a. adenocarcinoma b. islet cell tumor c. cystadenocarcinoma d. lymphangioma e. pancreaticoblastoma

b. islet cell tumor

You have been asked to perform a liver sonogram on a patient with AIDS. Which of the following tumors is most commonly associated with this history? a. hepatocellular carcinoma b. kaposi's sarcoma c. budd-chiari syndrome d. hemangiosarcoma e. hepatic adenoma

b. kaposi's sarcoma

Identify the vessel labeled "B" a. right hepatic vein b. left hepatic vein c. middle hepatic vein d. main portal vein e. left portal vein

b. left hepatic vein

Which of the following describes the most common patient positioning for performance of a transrectal prostate ultrasound? a. supine b. left lateral decubitus c. right oblique d. supine in reversed trendelenburg position e. standing

b. left lateral decubitus

Letter "D" represents which of the following structures? a. right renal artery b. left renal artery c. superior mesenteric artery d. celiac trunk e. right renal vein

b. left renal artery

Identify the structure labeled A in this image of the liver a. ligamentum venosum b. ligamentum teres c. hepatoduodenal ligament d. coronary ligament e. glisson's capsule

b. ligamentum teres

What muscle is located posterior to each lobe of the thyroid? a. sternocleidomastoid b. longus coli c. sternohyoid d. sternothyroid e. both c and d

b. longus coli

You are scanning a patient with right flank pain and known polycystic kidney disease. You suspect the presence of hemorrhage within one of the renal cysts. What is the sonographic appearance of this finding? a. ultrasound cannot be used to detect hemorrhage within a renal cyst b. low-level echoes within the cyst c. multiple bright foci with posterior acoustic shadowing distal to the cyst d. solid appearing nodule with increased attenuation e. all of the above

b. low-level echoes within the cyst

This transverse image was obtained with the patient in a right oblique position and the transducer placed over the spleen. What abnormality is present? a. small bowel obstruction b. lymphadenopathy c. polycystic kidney disease d. accessory spleen e. hydronephrosis

b. lymphadenopathy

This transverse image was obtained with the patient in a right oblique position and the transducer placed over the spleen. What is present? a. small bowel obstruction b. lymphadenopathy c. polycystic kidney disease d. accessory spleen e. hydronephrosis

b. lymphadenopathy

Identification of what anatomic structure would most help a sonographer locate a contracted gallbladder? a. ligamentum teres b. main lobar fissure c. right hepatic vein d. ligamentum venosum e. coronary ligament

b. main lobar fissure

You are performing an abdominal ultrasound study and detect adilated, nontender gallbladder. What should you look for? a. right kidney hydronephrosis b. mass in the head of the pancreas c. mass in the posterior right lobe of the liver d. abdominal aortic aneurysm e. portal vein thrombosis

b. mass in the head of the pancreas

You detect the presence of free fluid in the space between the liver and right kidney. What is the name for this anatomic location? a. pouch of douglas b. morison's pouch c. cul de sac d. space of disse e. foramen of winslow

b. morison's pouch

Which of the following is usually diagnosed in early childhood or in utero? a. autosomal dominant polycystic disease b. multicystic dysplastic kidney c. acquired cystic kidney disease d. parapelvic cysts e. a and b

b. multicystic dysplastic kidney

You are attempting to image the head of the pancreas in a patient referred for abdominal sonography. How should you align the probe on the body to obtain a long-axis view of the head and body of the pancreas? a. obtain a midline oblique scan plane with the left side of the probe slightly cephalad compared to the right side of the probe b. obtain a midline transverse scan plane with the left side of the probe slightly caudad compared to the right side of the probe c. obtain a oblique scan plane to the right of midline, with the left side of the probe slightly cephalad compared to the right side of the probe d. obtain a midline sagittal scan plane with the probe angled toward the patients' right side e. obtain a sagittal scan plane with the left side of the probe slightly cephalad compared to the right side of the probe

b. obtain a midline transverse scan plane with the left side of the probe slightly caudad compared to the right side of the probe

You are performing an abdominal doppler studt and suspect the presence of median arcuate ligament syndrome. To rule out this condition you should: a. obtain baseline doppler values, give the patient a fatty meal, and rescan at 10, 20, and 30 minutes following the meal b. obtain doppler readings during both inspiration and expiration and in both the supine and upright positions c. obtain doppler readings at an angle of 45 degrees and then repeat at an angle of 60 and 90 degrees d. obtain baseline doppler readings and have the patient return in approximately two weeks for a comparison study e. obtain doppler readings from the proximal, mid, and distal aspect of the artery

b. obtain doppler readings during both inspiration and expiration and in both the supine and upright positions

WHat is the most useful criterion for assessment of nodal disease? a. since ultrasound cannot detect normal abdominal lymph nodes, whenever they are seen they are abnormal b. only lymph nodes measuring greater than 1.0 cm are considered to be abnormal c. whenever the hilum of the lymph node is echogenic, it is considered to be abnormal d. only abdominal lymph nodes which have an oval shape are considered to be abnormal e. only abdominal lymph nodes which show increased vascularity by color doppler are considered to be abnormal

b. only lymph nodes measuring greater than 1.0 cm are considered to be abnormal

You are performing an ultrasound examination on a patient with acute cholecystitis. Complications of acute cholecystitis that you should look for include all of the following EXCEPT: a. pancreatitis b. pancreatic carcinoma c. gallbladder perforation d. gangrenous cholecystitis e. emphysematous cholecystitis

b. pancreatic carcinoma

A patient has been referred for abdominal sonography with abdominal pain and increased serum amylase. You detect a fluid pocket in the left pararenal space. GIven these ultrasound findings and patient history, what is the most likely etiology of this mass? a. exophytic renal cyst b. pancreatic pseudocyst c. retroperitoneal hemorrhage d. lymphocele e. urinoma

b. pancreatic pseudocyst

You have detected a stone impacted in the distal common bile duct in a patient with jaundice and abdominal pain. You will tailor your exam to evaluate what complication of this condition? a. aortic aneurysm b. pancreatitis c. portal vein aneurysm d. portal hypertension e. choledochal cyst

b. pancreatitis

What is the ultrasound appearance of ureteropelvic junction obstruction? a. dilated ureter and collecting system to the level of the urinary bladder b. pelvicaliaectasis to the level of the junction of the renal pelvis and ureter c. dilated ureter whith normal intrarenal collecting system d. pelvicaliectasis to the level of the distal ureter e. ureteropelvic junction obstruction cannot be detected sonographically

b. pelvicaliaectasis to the level of the junction of the renal pelvis and ureter

During abdominal sonography on a 68 year old male, you suspect an exophytic mass arising from the stomach. Which technique is most useful in analyzing this mass? a. scan the area while the patient performs a Valsalva maneuver b. perform compression sonography c. scan the patient in both inspiration and expiration d. rescan the patient following ingestion of a fatty meal e. scan the patient in a Trendelenburg position

b. perform compression sonography

A patient has been referred for ultrasound evaluation of the left kidney following an automobile accident. What is the sonographic appearance of a subcapsular hematoma? a. free fluid in Morison's pouch b. perirenal fluid collection that flattens the underlying renal contour c. a linear defect that extends throughout the kidney d. an intrarenal fluid collection within the renal collecting system e. a subcapsular hematoma will not be visible by ultrasound

b. perirenal fluid collection that flattens the underlying renal contour

The kidneys lie in what retroperitoneal space? a. posterior pararenal space b. perirenal space c. retrofascial space d. anterior pararenal space e. the kidneys are not retroperitoneal in location

b. perirenal space

In what anatomic variant is the fundus of the gallbladder folded over the body? a. choledochal cyst b. phrygian cap c. duplicated collecting system d. biliary atresia e. junctional fold

b. phrygian cap

A recannalized paraumbilical vein may be seen as a result of which of the following? a. hepatic adenoma b. portal hypertension c. hepatitis d. hepatic trauma e. liver biopsy

b. portal hypertension

A 38-year-old female has been referred for a Doppler study of the liver with a history of ETOH abuse and cirrhosis. What vascular condition is associated with this history? a. mesenteric ischemia b. portal hypertension c. hepatic artery fibromuscular dysplasia d. splenic artery pseudoaneurysm e. Budd-chiari syndrome

b. portal hypertension (most common cause of portal hypertension is cirrhosis from alcohol abuse)

The majority of blood supply to the liver is provided from the: a. hepatic veins b. portal vein c. hepatic artery d. SMV e. gastroduodenal artery

b. portal vein

You are having difficulty identifying the pancreas in an elderly patient. You can identify the splenic vein. What is the relationship of the splenic vein to the pancreas? a. posterior and superior b. posterior and inferior c. anterior and superior d. anterior and inferior e. the splenic vein does not approximate the pancreas

b. posterior and inferior

Which of the following describes the anatomic location of the right diaphragmatic crus? a. anterior to the IVC and superior to the right renal artery b. posterior to the IVC and right renal artery c. medial to the right kidney and posterior to the abdominal aorta d. anterior to the abdominal aorta and inferior to the pancreas e. lateral to the pancreatic head and anterior to the IVC

b. posterior to the IVC and right renal artery

What is the relationship of the superior mesenteric artery to the pancreas? a. posterior to the tail b. posterior to the neck c. superior to the body d. cephalad to the head e. lateral to the tail

b. posterior to the neck

A patient has been referred for a doppler study with symptoms of chronic mesenteric ischemia. What are the symptoms of this disease? a. weight gain, abdominal gas, and bloating b. postprandial abdominal pain and weight loss c. flank pain radiating toward the hip d. right upper quadrant radiating to the left shoulder e. hypertension, weight gain, and abdominal pain

b. postprandial abdominal pain and weight loss

You are performing an abdominal sonogram and detect a striated, hypoechoic structure immediately posterior to the right kidney. A similar structure is seen posterior to the left kidney. What are you imaging? a. psoas muscle b. quadratus lumborum muscle c. adrenal gland d. diaphragmatic crura e. gerota's fascia

b. quadratus lumborum muscle

Which of the following renal tumors is more common? a. transitional cell carcinoma b. renal cell carcinoma c. oncocytoma d. primary renal lymphoma e. angiomyolipoma

b. renal cell carcinoma

What doppler parameter should you measure to look for rejection in a renal transplant? a. pulsatility index b. resistivity index c. renal-aortic ratio d. systolic-diastolic ratio e. acceleration index

b. resistivity index

You are scanning the urinary bladder and notice multiple artifactual bands in the near field at the anterior bladder wall. What is the source of these echoes? a. acoustic speckle b. reverberation artifact c. comet tail artifact d. mirror image artifact e.multipath artifact

b. reverberation artifact

The inferior vena cava empties blood into which of the following? a. abdominal aorta b. right atrium c. left atrium d. right ventricle e. left ventricle

b. right atrium

Identify the anatomy labeled A. a. main portal vein b. right posterior portal vein branch c. left portal vein d. right hepatic vein e. middle hepatic vein

b. right posterior portal vein branch

Identify the anatomy labeled "D" a. common hepatic artery b. splenic artery c. left renal vein d. left renal artery e. inferior mesenteric artery

c. left renal vein

Ultrasound findings of an abdominal study on a 51-year-old female include enlargement of the hepatic veins and inferior vena cava in an otherwise normal appearing liver. These findings are most consistent with which of the following? a. Budd-chiari syndrome b. right-sided heart failure c. liver cirrhosis d. portal hypertension e. sarcoidosis

b. right-sided heart failure

During performance of a renal sonogram, you identify only one kidney. What should you do? a. inform the patient of your finding and advise a thourough medical exam to detect other anatomic anomalies b. scan in the pelvis area to rule out the presence of a pelvic kidney c. perform an endovaginal exam to look for bicornuate uterus d. scan the patient in an upright position e. perform a complete doppler study of the solitary kidney

b. scan in the pelvis area to rule out the presence of a pelvic kidney

Which of the following transduceres does NOT demonstrate a sector view? a. phased array b. sequential linear array c. curved linear array d. annular array e. all of these transducers produce a sector field of view

b. sequential linear array

You are scanning a patient with suspected liver cirrhosis. All of the following are sonographic features of cirrhosis EXCEPT: a.surface nodularity b. shrunken caudate lobe c. altered echo texture d. ascites e. regenerative nodules

b. shrunken caudate lobe ( with cirrhosis the caudate is usually enlarged compared to the right due to sparing)

A patient with a history of chronic medical renal disease has been referred for abdominal ultrasound. Which of the following describes the renal appearance you expect to see? a. enlarged hypoechoic kidneys b. small hyperechoic kidneys c. normal appearance of kidneys d. small hypoechoic kidneys e. normal sized kidneys with calcified collecting system

b. small hyperechoic kidneys

A 49-year-old male presented with a clinical history of liver cirrhosis and portal hypertension. In the transverse image below, multiple hypoechoic structures are seen at the splenic hilum and between the kidney and spleen. What is the most likely etiology of these structures. a. multiple aneurysms of the splenic artery b. splenic and retroperitoneal varices c. loculated ascites d. polycystic kidney disease e. fluid-filled loops of bowel

b. splenic and retroperitoneal varices

Which visceral artery is most commonly involved with aneurysmal formation? a. hepatic artery b. splenic artery c. SMA d. IMA e. gastroduodenal artery

b. splenic artery

You are performing an abdominal ultrasound scan and detect a hypoechoic, wedge-shaped lesion within the spleen. This is the typical appearance of which of the following entities? a. splenic hemangioma b. splenic infarction c. splenic metastasis d. granulomatous disease of the spleen e. splenic abscess

b. splenic infarction

Which structures will you detect at the splenic hilum? a. diaphragm b. splenic vein c. left kidney d. duodenum e. left lobe of the liver

b. splenic vein

You have been asked to rule out the presence of choledocholithiasis. What are you looking for? a. inflammation with thickening of the gallbladder wall b. stones within the common bile duct c. calcified gallbladder wall d. contracted gallbladder filled with stones e. gallbladder carcinoma

b. stones within the common bile duct

During insonation of the pancreas, you routinely image a vessel anterior to the uncinate process. What is this vessel? a. inferior mesenteric vein b. superior mesenteric vein c. gastroduodenal artery d. left renal vein e. splenic artery

b. superior mesenteric vein

A patient has been referred for ultrasound evaluation of tendinitis in the wrist. What is the ultrasound appearnce of a tendon? a. the songoraphic appearance of a tendon caries greatly by location within the body b. tendons are highly echogenic with a fibrillar echotexture c. tendons are markedly hypoechoic and homogeneous d. tendons have a heterogeneous sonographic apperance with mixed echodensity e. tendons cannot be recognized sonographically

b. tendons are highly echogenic with a fibrillar echotexture

Which of the following statements is true regarding measurements obtained from this doppler waveform? a. the end diastolic velocity will be overestimated b. the peak systolic velocity will be underestimated c. the peak systolic velocity will be overestimated d. the resistive index will be accurate e. all of the above

b. the peak systolic velocity will be underestimated

Which of the following describes the most widely used conventional display of the prostate using transrectal sonography? a. the anterior abdominal wall is shown at the bottom of the screen b. the rectum is shown at the bottom of the screen c. the rectum is shown to the right of the screen in a sagittal view d. the left side of the patient is shown on the left side of the image in a transverse plane e. the head of the patient is on the right side of the image in a sagittal plane

b. the rectum is shown at the bottom of the screen

Which of the following statements is NOT true regarding the spleen? a. the spleen is located in the left upper quadrant b. the spleen is a retroperitoneal structure c. the spleen lies between the fundus of the stomach and the diaphragm d. the average adult spleen measures 12 cm in length e. the normal spleen decreases in size and weight with advancing age

b. the spleen is a retroperitoneal structure (the spleen is located intraperitoneally)

You are performing an ultrasound of a patient with a known "bell clapper" deformity. This deformity is associated with which condition? a. seminoma b. torsion c. varicocele d. epididymitis e. infarction

b. torsion

Identify the structure labeled C a. esophagus b. trachea c. sternohyoid d. cervical spine e. jugular vein

b. trachea

Which of the following describes the normal flow pattern in the hepatic veins? a. biphasic b. triphasic c. monophasic d. continuous e. it is normal to see any of the above waveform types in the hepatic veins

b. triphasic

During a routine abdomen and pelvic ultrasound study, you detect a small round, cystic structure projecting into the urinary bladder. This most likely represents: a. urinoma b. urterocele c. transitional cell carcinoma d. papillary necrosis e. extrarenal pelvis

b. urterocele

For correct orientation when performing a transverse scan through the left thyroid, which letter represents the patient's right side? a. a b. b c. c d. d e. e

c

A thin patient has been referred for ultrasound of the pancreas. Which of the following transducers would be best for this examination? a. 2.25 MHz phased linear array b. 3.2 MHz curved linear array c. 5.0 MHz curved linear array d. 7.5 MHz small footprint phased array e. 12 MHz linear array

c. 5.0 MHz curved linear array

An average sized male patient has been referred for ultrasound evaluation of splenomegaly. What transducer would provide the highest resolution images of the spleen? a. 2.25 MHz linear array b. 3.5 MHz phased array c. 5.0 MHz curved linear array d. 3.5 MHz curved linear array e. 2.25 MHz phased array

c. 5.0 MHz curved linear array

You are performing a doppler study on a patient with a TIPS stent. What is the greatest doppler angle of incidence you should use to get accurate velocity measurements in this exam? a. 0 degrees b. 45 degrees c. 60 degrees d. 70 degrees e. 90 degrees

c. 60 degrees

For correct orientation when performing a sagittal scan in the right breast, which letter is on the side of the image toward the patient's head? a. A b. B c. C d. D e. E

c. C

You have been asked to aid in staging of pancreatic cancer. Which procedure is most accurate in staging pancreatic adenocarcinoma? a. abdominal ultrasound b. endoscopic ultrasound c. CT d. MRI e. plain film x-ray

c. CT

Which vessel is located superior to the vessel labeled "E"? a. SMA b. IMA c. Celiac trunk d. left renal vein e. right renal artery

c. Celiac trunk

You have performed an ultrasound study on a patient with an enlarged caudate lobe, shrunken right lobe and splenomegaly. The hepatic veins could not be identified. No other abnormalities were discovered. What should you do? a. scan the pelvis to rule out a pelvis mass b. have the patient perform a valsalva maneuver and reexamine the hepatic veins c. evaluate the hepatic veins and IVC with color doppler to confirm patency d. have the patient return in a week for a repeat study to evaluate the hepatic veins e. nothing, you have completed the exam

c. Evaluate the hepatic veins and IVC with color doppler to confirm patency

The hepatic veins drain into what vessel? a. main portal vein b. splenic vein c. IVC d. hepatic artery e. caudate vein

c. IVC

Which of the following describes the best sonographic window to view a recannalized paraumbilical vein? a. intercostal oblique view through the right lobe b. subcostal oblique view through the right lobe c. sagittal subcostal view through the left lobe at the level of the ligamentum teres d. sagittal subcostal view through the right lobe at the level of the main lobar fissure e. sagittal subcostal veiw to the left of midline

c. Sagittal subcostal view through the left lobe at the level of the ligamentum teres

You are performing a liver sonogram on a young hemale with RUQ pain, sudden onset ascites and hepatomegaly. You have obtained transverse and sagittal images of the liver, common bile duct, and gallbladder according to your protocol. What else should you do? a. nothing, the study is complete b. expand the study to include the kidneys to rule out associated hydronephrosis c. use color and spectral doppler to determine patency of the portal and hepatic venous systems d. give the patient a fatty meal and then measure the portal vein diameter at 1,2,5, and 10 minutes e. call the referring physician to get an order to perform a pelvic study to see if the patient's pain is referred from an ovarian mass

c. Use color and spectral doppler todetermine patency of the portal and hepatic venous systems

Which of the following describes the best view for ultrasound demonstration of the coronary vein? a. a transverse scan under the right lobe of the liver b. an oblique subcostal scan under the right lobe of the liver with the probe oriented toward the patient's head c. a sagittal view of the splenic vein near the midline d. a sagittal view throught the splenic hilum e. a transverse view along the long axis of the left renal vein

c. a sagittal view of the splenic vein near the midline (coronary vein drains into the splenic vein near the level of the portal splenic confluence)

Which of the following is NOT true regarding thyroid nodules? a. the overwhelming majority of thyroid nodules are benign b. thyroid nodules are very common c. a solitary thyroid nodule is usually malignant d. nodules with significant cystic component are usually benign e. nodules may be hyperechoic or hypoechoic to the thyroid

c. a solitary thyroid nodule is usually malignant

You are scanning a 34-year-old multiparois woman with symptoms of severe RUQ pain, nausea and vomiting. The gallbladder is thick-walled with stones and an adjacent complex fluid collection is seen. These findings most likely represent: a. adenomyomatosis complicated by gallstones b. gallbladder carcinoma c. acute cholecystitis complicated by gallbladder perforation d. acalculous cholecystitis e. emphysematous cholecystitis

c. acute cholecystitis complicated by gallbladder perforation

You are performing an ultrasound study to rule out the presence of a pancreatic tumor. What is the most commonly occuring malignant tumor of the pancreas? a. cystadenocarcinoma b. insulinoma c. adenocarcinoma d. gastrinoma e. Klatskin tumor

c. adenocarcinoma

You have been asked to perform graded compression sonography of the appendix in a young patient with questionable acute appendicitis. Which of the following describes the technique you will employ? a. perform quick, short compressions in the right lower quadrant to assess pain and size of the appendix b. quickly push the probe as far as possible into the abdomen and hold long enough to image the appendix and surrounding tissue c. apply gradual and uniform pressure with the probe over the area of interest d. push slowly and uniformly with the hand just superior to the probe to push the tissue into the field of view e. any of the above maneuvers can be used

c. apply gradual and uniform pressure with the probe over the area of interest

What arteries course on top of the renal pyramids and give rise to the tiny intralobular arteries? a. segmental b. interlobar c. arcuate d. vasa recta e. capsular

c. arcuate

What arteries course on top of the renal pyramids and give rise to the tiny intralobular arteries? a. segmental b. interlobar c. arcuate d. vasa recta e. capsular

c. arcuate

Which of the following describes the normal waveform of the main renal artery? a. triphasic b. high resistance c. low resistance d. phasic e. bidirectional

c. low resistance

A patient has been referred for ultrasond evaluation to rule out the presence of a carotid body tumor. Where is this tumor located? a. at the origin of the common carotid on each side of the neck b. lateral to the carotid bifurcation and medial to the sternocleidomastoid muscle c. at the level of the carotid bifurcation between the internal and external carotid arteries d. superomedial to the submandibular gland on each side of the neck e. within the sternocleidomastoid muscle, lateral to the carotid bifurcation

c. at the level of the carotid bifurcation between the internal and external carotid arteries

You have been asked to identify the ureteral "jets". Where are the ureteral orifices in the urinary bladder? a. each lateral edge b. superior and anterior border c. base of the trigone along posterior aspect d. inferior and anterior to the trigone e. at the bladder base, inferiorly at the apex of the trigone

c. base of the trigone along posterior aspect

What is the most common cause of acute cholecystitis? a. hepatitis b. gallstone lodged in the fundus of the gallbladder c. calculus obstruction of the gallbladder neck or cystic duct d. pancreatitis e. hepatocellular carcinoma

c. calculus obstruction of the gallbladder neck or cystic duct

What lobe of the liver does the letter c represent? a. left lobe b. caudate lobe c. posterior right lobe d. anterior right lobe e. quadrate lobe

c. caudate lobe

You have idenified a single homogenous hyperechoic lesion measuring 2.4 cm in the posterior aspect of the right lobe of the liver. What is the most common etiology of a mass fitting this description? a. cyst b. hepatic adenoma c. cavernous hemangioma d. hepatocellular carcinoma e. focal fatty sparing

c. cavernous hemangioma

Which of the following is NOT a sign of acalculous cholecystitis? a. gallbladder wall thickening b. sonographic Murphy's sign c. cholelithiasis d. pericholecystic fluid e. gallbladder wall edema

c. cholelithiasis

You are scanning through a liver and notice luminal narrowing of the hepatic veins. Color and spectral doppler reveal high velocities through the strictures. These findings are most commonly associated with which of the following? a. diffuse fatty liver b. acute hepatits c. cirrhosis d. focal fatty infiltration e. glycogen storage disease

c. cirrhosis (as cirrhosis progresses the thin walls of the hepatic veins compress)

The image was obtained from the left upper quadrant of a patient with abdominal pain and bloating. What is the structure seen anterior and inferior to the left kidney? a. small intestine b. stomach c. colon d. retroperitoneal fibrosis e. lymphadenopath

c. colon

You are having difficulty obtaining an adequate color doppler image of a deep abdominal vessel. What color doppler prameter will increase the signal-to-noise (SNR) ratio and result in a stronger signal? a. color filter b. color pulse repetition frequency c. color packet size d. color baseline e. color reject

c. color packet size

You are imaging an arery in a patient with a history of previous surgery and detect surgical clips. What artifacts is seen posterior to the surgical clips in this image? a. refraction b. ringdown c. comet tail d. enhancement e. reverberation

c. comet tail

What vessel is anterior to the uncinate process and pasterior to the neck of the pancreas? a. superior mesenteric artery b. inferior mesenteric artery c. confluence of portal and splenic vein d. inferior mesenteric vein e. common hepatic artery

c. confluence of portal and splenic vein

This image was obtained in what plane? a. transverse b. sagittal c. coronal d. oblique e. none of the above

c. coronal

You are performing a doppler study of the hepatic venous system and cannot obtain a color doppler signal from the main portal vein. What should you do? a. increase the color dynamic range b. increase the system high pass filter c. decrease the system pulse repetition frequency d. increase the doppler angle of incidence e. decrease the color gain

c. decrease the system pulse repetition frequency

You are scanning through the right lobe of the liver and notice that although you have maximized the far field TGC, the parenchyma in the far field and diaphragm are not clearly visualized. What should you do? a. decrease the transmit power b. increase the compression curve c. decrease the transmit frequency d. decrease the overall gain e. increase the dynamic range

c. decrease the transmit frequency

A patient has been referred for transrectal sonography due to symptoms of prostatism. What are the most common patient symptoms of prostatism? a. nausea, painful urination, pressure sensation on bladder b. weight loss, nocturia, hematuria, and small stream c. difficult initiation of voiding, nocturia, and small stream d. hematuria, dysuria, and nocturia e. swelling, lethargy, dysuria, and nocturia

c. difficult initiation of voiding, nocturia, and small stream

A patient has been referred for transrectal sonography due to symptoms of prostatism.WHat are the most commmon symptoms of prostatism? a. neausea, painful urination, pressure sensation on urinary bladder b. weight loss, nocturia, hematura, and small stream c. difficult initiation of voiding, nocturia, and small stream d. hematuria, dysuria, and nocturia e. swelling, lethargy, dysuria, and nocturia

c. difficult initiation of voiding, nocturia, and small stream

You have been asked to perform an ultrasound evaluation of a child with multicystic dysplastic kidney (MCDK). Which of the following is NOT a sign of this condition? a. multiple variably sized cysts b. nonmedial lovation of the largest cyst c. dilated ureter d. no identifiable renal sinus e. brightly echogenic tissue interfaces between cysts

c. dilated ureter

During a renal sonogram, you notice a 1.5 cm thickening of the left lateral renal cortex. This most likely represents: a. column of Bertin b. angiomyolipoma c. dromedary hump d. medullary pyramid e. hilar vessels

c. dromedary hump

You are scanning a patient in the emergency room with severe nausea and abdominal pain. When you press with the transducer over the gallbladder ,the patient has increased pain. What did you do? a. performed a valsalva maneuver b. performed a trendelenburg maneuver c. elicted a sonographic Murphy's sign d. elicted a Homan's sign e. elected a Morison's response

c. elicted a sonographic Murphy's sign

You have been requested to perform a transrectal ultrasound of the prostate in a patient with suspected benign prostatic hyperplasia (BPH). What is the ultrasound apperance of this condition? a. small, shrunken prostate gland with numerous calcifications b. small prostate gland with increased vascularity and hypoechoic nodules c. enlarged prostate gland which may be diffuse or focal d. enlargement of the peripheral zone with sparing of the inner gland e. all of the above appearances are commonly encountered with BPH

c. enlarged prostate gland which may be diffuse or focal

You detect a wedge-shaped, hypoechoic lesion within the spleen. Which of the following would increase diagnostic confidence the most? a. have the patient perform a valsalva maneuver b. evaluate the lesion in both inspiration and expiration c. evaluate the lesion with color doppler d. have the patient drink 48 o of water and rescan e. rescan the patient in an upright position

c. evaluate the lesion with color doppler ( the spleen is highly vacular, an infarct would show no color and would stand out from it's surrounding vascularity)

Which of the following would be most helpful in delineating ureteral "jets"? a. have patinet perform valsalva maneuver b. scan patient in both inspiration and expiration c. examine the urinary bladder with color doppler d. increase the transducer frequency e. give the patient a fatty meal

c. examine the urinary bladder with color doppler

The superior thyroid artery is a branch of what artery? a. common carotid b. internal carotid c. external carotid d. thyrocervial e. inferior thyroid

c. external carotid

Which of the following is NOT a direct branch of the abdominal aorta? a. celiac trunk b. SMA c. external iliac artery d. IMA e. renal artery

c. external iliac artery

A liver ultrasound on a 49-year-old obese male demonstrates diffuse increased echogenicity with a focal hypoechoic area anterior to the portal vein. This most likely represents: a. liver cirrhosis with hepatocellular carcinoma b. hydatid disease c. fatty metamorphosis of the liver with focal sparing d. metastatic disease most likely due to a colon primary e. normal liver parenchyma with a simple cyst

c. fatty metamorphosis of the liver with focal sparing

You are performing an ultrasound exam on a young female and notice a well-defined solitary mass with a central scar measuring 4 cm in diameter. Color doppler reveals prominent blood vessels coursing within the scar. This most likely represents a. liver abscess b. hepatic adenoma c. focal nodular hyperplasia d. hepatocellular carcinoma e. liver metastasis from lung cancer

c. focal nodular hyperplasia ( 2nd most common benign liver mass, behind hemangioma. More common in women of childbearing age. Central scar and peripheral and central vascularity)

What structure can you use to identify the anterior aspect of the head of the pancreas? a. common bile duct b. common hepatic artery c. gastroduodenal artery d. splenic vein e. left renal vein

c. gastroduodenal artery

Which structure can be seen coursing transversely at the level of the upper pancreatic head? a. common bile duct b. common hepatic artery c. gastroduodenal artery d. splenic vein e. left renal vein

c. gastroduodenal artery

You are performing a thyroid ultrasound on a young woman with hypothyroidism. You detect a moderately enlarged, hypoechoic thyroid witha coarseded parenchymal echo texture. Which of the following conditions is most likely present, considering this history and findings? a. grave's disease b. multinodular adenomatous goiter c. hashimoto's thyroiditis d. papillary carcinoma e. normal thyroid

c. hashimoto's thyroiditis

During transverse imaging of the thyroid, you detect a hypoechoic, round nodule posterior and medial to the thyroid. How can you determine if this is the eshophagus or a nodule? a. image the patient in both inspiration and expiration b. have the patient perform valsalva maneuver c. have the patient swallow while scanning the area d. scan the patient in an upright position e. all of the above

c. have the patient swallow while scanning the area

A single large, well-defined mass wiht smooth walls and homogeneous low-level echoes is seen within the aneterior right lobe of the liver 48-tear-old female. No doppler signals could be obtained within the mass. Which of the following conditions is the most likely etiology of this mass? a. Kaposi's sarcoma b. focal nodular hyperplasia c. hemorrhagic cyst d. portal vein aneurysm e. hepatocellular carcinoma

c. hemorrhagic cyst

A patient is referred for a liver ultrasound with the clinical history of a raised serum alpha-fetoprotein level. What should you look for? a. focal nodular hyperplasia b. fatty liver c. hepatocellular carcinoma d. hydatid disease e. increased alpha-fetoprotein levels are not associated with liver disease

c. hepatocellular carcinoma

While performing a doppler evaluation of the abdominal vessels, you detect a stenosis in the right renal artery. Which of the following is NOT a sign of stenosis with spectral analysis? a. spectral broadening distal to the stenosis b. increased peak systolic velocity at the stenosis c. increased peak diastolic velocity proximal to the stenosis d. increased pulsatility proximal to the stenosis e. dampening of the waveform distal to the stenosis

c. increased peak diastolic velocity proximal to the stenosis

You are performing an ultrasound on an obese patient and notice a small, hypoechoic tumor located in the tail of the pancreas. This most likely represents: a. adenocarcinoma b. cystadenocarcinoma c. insulinoma d. Klatskin tumor e. pancreaticblastoma

c. insulinoma

You are having difficulty locating the gallbladder in a patient with RUQ pain. What anatomic landmark will help you identify the gallbladder fossa? a. ligamentum venosum b. ligamentum teres c. interlobar hepatic fissure d. falciform ligament e. coronary ligaments

c. interlobar hepatic fissure

You have been asked to identify the left adrenal gland. WHich describes the correct anatomic relationship of this gland? a. superolateral to the left kidney b. anterior to the pancreatic tail c. lateral to the abdominal aorta and diaphragmatic crus d. inferior to the quadratus lumborum muscle e. inferior and lateral to the splenic hilum

c. lateral to the abdominal aorta and diaphragmatic crus

Identify the anatomy labeled C a. main portal vein b. right posterior portal vein c. left portal vein d. right hepatic vein e. middle hepatic vein

c. left portal vein

While performing an abdominal ultrasound, you detect multiple hypoechoic nodules surrounding the abdominal aorta and separating the mesenteric vessels .You are most likely imageing: a. pancreatic pseudocyst b. retroperitoneal fibrosis c. lymphadenopathy d. urinomas e. lymphoceles

c. lymphadenopathy

What is the sonographic appearance of a normal peripheral nerve? a. markedly hypoechoic structure with scattered bright internal foci b. striated, hypoechoic, linear structure c. markedly hyperechoic structure with parallel internal linear echoes d. homogeneous, hyperechoic, structure with barbell shape e. periperal nerves cannot be detected sonographically

c. markedly hyperechoic structure with parallel internal linear echoes

The rete teste is located in the : a. pampiniform plexus b. spermatic cord c. mediastinum testis d. appendix testis e. appendix epididymis

c. mediastinum testis

What pathology is present in this image? a. spermatocele b. seminoma c. microlithiasis d. metastatic disease e. orchitis

c. microlithiasis

Identify the vessel labeled "A" a. right hepatic vein b. left hepatic vein c. middle hepatic vein d. main portal vein e. left portal vein

c. middle hepatic vein

Which vessel courses within the main lobar fissure? a. main portal vein b. left portal vein c. middle hepatic vein d. proper hepatic vein e. right hepatic vein

c. middle hepatic vein

You have been asked to perform a doppler analysis of the portal venous system. Which of the following describes the waveform you will see in the portal vein in a normal study? a. bidirectional b. triphasic c. mildly undulating d. highly pulsatile e. continuous

c. mildly undulating (smooth wavelike motion and low velocity is normal)

Identify the etiology of the structure indicated by the small arrows in this image: a. cystic mass in the right lung b. loculated pleural effusion c. mirror image of a liver cyst d. ectopic location of the gallbladder e. pleural abscess

c. mirror image of a liver cyst

You are scanning a patient with suspected lymphoma of kidney. Which ultrasound appearance is associated with renal lymphoma? a. small,echogenic kidneys with hyperdense pyramids b. cystic masses of varying sizes throughout both kidneys c. multiple, bilateral, hypoechoic masses in enlarged kidneys d. single, large, hyperechoic mass e. unilateral wedge-shaped hypoechoic mass

c. multiple, bilateral, hypoechoic masses in enlarged kidneys

You are performing an ultrasound on a patient with a recent aortoiliac graft and detect this waveform adjacent to the graft site near a pulsating hematoma. This waveform was most likely obtained from which of the following? a. graft stenosis b. graft occlusion c. neck of pseudoaneurysm d. normal graft e. graft aneurysm

c. neck of pseudoaneurysm (pseudoaneurysm has a high velocity, bi-directional "to and fro" signal)

You detect a hypoechoic region within an otherwise normal appearing tendon. What causes false hypoechogenicity within a tendon? a. use of too much scan gel between the skin and transducer surface b. scanning with the beam perpendicular to the tendon axis c. oblique incidence of the beam to the tendon axis d. use of a transducer frequency greater than 10 MHz e. overuse of receiver gain

c. oblique incidence of the beam to the tendon axis

Ultrasound findings in a patient with hypertension include a left kidney measuring 6.8 cm and a right kidney measuring 11.7 cm. Which of the following is most consistant with these findings? a. acute pyelonephritis in the left kidney b. acute glomerulonephritis in the right kidney c. occlusion of the left main renal artery d. amyloidosis of the right kidney e. none of the above

c. occlusion of the left main renal artery

Which of the following conditions would show hyperemic flow on color doppler? a. testicular torsion b. spermatocele c. orchitis d. hydrocele e. infarction

c. orchitis

This image was obtained in a 48-year-old male referred for an abdominal ultrasound because of abnormal liver function tests. Which of the following best describes the findings? a. ectopic location of gallbladder b. pancreatic head carcinoma c. pancreatic head pseudocyst d. fluid in the duodenum e. peripancreatic lymphadenopathy

c. pancreatic head pseudocyst

During sonographic evaluation of the periaortic area, you detect several hypoechoic structures each measuring slightly greater than 2.0 cm, adjacent to the celiac trunk and SMA. WHat is the most likely etiology of these structures? a. diaphragmatic crus b. retroperitoneal fibrosis c. para-aortic lymph nodes d. mesenteric aneurysms e. lymphoceles

c. para-aortic lymph nodes

You are scanning a 31-ear-old patient with hypertension and impaired renal function. You detect enlarged kidneys with cysts that are too numerous to count. Which of the following is most likely? a. multicystic dysplastic kidney b. medullary cystic disease c. polycystic kidney disease d. parapelvic cysts e. multiple simple cysts

c. polycystic kidney disease (the only option that would result in enlargement of the kidney)

You are performing a follow-up study on a patient with a history of cavernous transformation. Where should you look to evaluate this condition? a. splenic hilum b. pancreatic head c. porta hepatis d. renal hilum e. left lobe of the liver

c. porta hepatis (cavernous transformation may occur following portal vein thrombosus. Appears as multiple serpiginous channels in the porta hepatis where the portal vein was located)

Hepatofugal flow in the portal vein is a sign of: a. normality b. hepatic artery thrombosis c. portal hypertension d. acute cholecystitis e. hepatocellular carcinoma

c. portal hypertension

The splenic vein drains into what vein? a. inferior mesenteric vein b. superior mesenteric vein c. portal vein d. left renal vein e. inferior vena cava

c. portal vein

Which of the following best describes the location of the distal common bile duct? a. anterior and superior to the pancreatic tail b. medial and caudal to the pancreatic neck c. posterior and slightly lateral to the pancreatic head d. inferior and medial to the pancreatic neck e. posterior and medial to the pancreatic head

c. posterior and slightly lateral to the pancreatic head

Which of the following would be an indicator of pancreatic transplant rejection? a. high-resistance doppler signals b. low-resistance doppler signals c. heterogenous parenchyma d. a and c e. b and c

d. a and c

The mediastinum testis is identified sonographically as: a. hypoechoic structure coursing throuh midline of testis b. pyramid shaped structure adjacent to the upper pole of the testis c. prominent echogenic linear echo in the midline of the testis d. homogeneous ovoid structure adjacent to the lower pole of the testis e. cystic, spherical structure of variable location

c. prominent echogenic linear echo in the midline of the testis

During ultrasound evaluation of the kidney, you detect a striated structure located posteromedial in relation to the kidney. What muscle are you imaging? a. quadratus lumborum b. rectus sheath c. psoas d. iliacus e. piriformis

c. psoas

This TRV image was obtained in a 49 y.o. male with hematuria. What structure is represented by the letter "a"? a. rib b. lumbar spine c. quadratus lumborum muscle d. psoas muscle e. obturator internus muscle

c. quadratus lumborum muscle

You are imaging the gallbladder in a transverse orientation and notice a long shadow at both edges. What is the etiology of this shadow? a. normal shadowing from the cystic duct b. small stones within the gallbladder c. refraction artifact d. volume averaging artifact e. slice thickness artifact

c. refraction artifact

You are performing a sonogram on a slender female and notice a long, thin extension of the inferior aspect of the right lobe of the liver. This most likely represents: a. caudate lobe b. quadrate lobe c. reidel's lobe d. accessory liver e. papillary projection of the caudate lobe

c. reidel's lobe

A 54-year-old man has been referred for an abdominal doppler ultrasound with a history of uncontrollable high blood pressure. What pathology are you searching for? a. vascular compromise of the sma, ima, and celiac trunk b. hepatofugal portal vein flow c. renal artery stenosis d. splenic varices e. aortic aneurysm

c. renal artery stenosis (renal artery stenosis of 60% or greater can result in hypertension)

What pathology is seen in this image of the left kidney? a. angiomyolipoma b. renal cell carcinoma c. renal calculi d. left adrenal pheochromocytoma e. there is no pathology demonstrated on this image

c. renal calculi

What is the arrow pointing to in this image? a. mirror image artifact b. comet tail artifact c. reverberation artifact d. excessive gain e. side lobe artifact

c. reverberation artifact

What is the vessel labeled C? a. right renal vein b. left renal vein c. right renal artery d. left renal artery e. superior mesenteric artery

c. right renal artery

Which of the following vessels courses posterior to the IVC? a. right hepatic artery b. left renal vein c. right renal artery d. left renal artery e. none of the above

c. right renal artery

A patient presents for abdominal ultrasound with a history of jaundice, weight loss, and nausea. You detect dilatation of the CBD at the level of the porta hepatis but are unable to see the distal CBD due to overlying bowel gas. Which of the following would be most helpful in imporving visualization of the CBD? a. place the patient in a trendelenburg position b. have the patient cough several times c. roll the patient into a right posterior oblique position d. roll the patient into a prone position e. roll the patient into a left lateral decubitus position

c. roll the patient into a right posterior oblique position (RLD is most helpful in visualizing CBD without bowel gas interferance, LLD is helpful in demonstrating gallbladder or more proximal CBD but doesn't help demonstrate the distal or pancreatic portion of the duct)

You are performing a follow-up ultrasound in a patient with a known AAA. You cannot obtain a clear image of the aorta in a midline sagittal view due to overlying bowel gas. What should you do? a. have the patient return after fasting overnight b. have the patient perform a valsalva maneuver while scanning the same area c. roll the patient into either decubitus position and image the aorta from a coronal plane d. scan the patient in a reversed trendelenburg position e. scan the aorta from a prone position

c. roll the patient into either decubitus position and image the aorta from a coronal plane

You have performed a study to rule out appendicitis in a young female with RLQ pain. No evidence suggestin appendicitis was detected. What should you do? a. nothing the study is complete b. suggest the patient have a barium enema c. search for other causes of right lower quadrant pain d. have the patient return in 24-48 hours for a repeat study if symptoms have not subsided e. both b and d

c. search for other causes of right lower quadrant pain

You are performing an ultrasound exam in a patient with a history of alcoholic liver cirrhosis. You have documented the presence of splenomegaly and dilated veins at the splenic hilum. Considering the patient's history and findings, what else should you look for? a. search for signs of acute cholecystitis b. carefully scan the spleen for presence of infarcts c. search for the presence of portosystemic collaterals d. check the pelvis for a left side mass e. rule out the presence of an aortic aneurysm

c. search for the presence of portosystemic collaterals

Which lab values would be of greatest benefit to you in evaluation of a patient with acute pancreatitis? a. alphafetoprotein b. serum creatinine c. serum amylase d. serum albumin e. alkaline phosphatase

c. serum amylase

Which part of the kidney contains fat, calyces, infundibuli of the collecting system, and vessels? a. medulla b. cortex c. sinus d. pyramid e. gerota's fascia

c. sinus

Identify the structure labeled A a. strap muscle b.longus coli muscle c. sternocleidomastoid muscle d. esophagus e. isthmus

c. sternocleidomastoid muscle

Which artery supplies the small intestine, right colon and most of the transverse colon? a. celiac trunk b. splenic artery c. superior mesenteric d. inferior mesenteric e. left gastric

c. superior mesenteric

You are attempting to locate the common hepatic duct at the porta hepatis. What is the most common anatomic relationship of the portal triad at this location? a. the common duct is posterior to the hepatic artery and anterior to the portal vein b. the common duct is anterior to the hepatic artery and posterior to the portal vein c. the common duct is anterior to the hepatic artery and portal vein d. the common duct is posterior to the hepatic artery and portal vein e. the common duct bears no relation to the hepatic artery and portal vein

c. the common duct is anterior to the hepatic artery and portal vein

You are scanning a patient with a porcelain gallbladder .What does this term mean? a. the gb wall is asymmetrically thickened b. the gb is enlarged and tender c. the gb wall contains varying amounts of calcification d. the gb is enlarged and nontender e. the gb contains multiple small cholesterol polyps

c. the gb wall contains varying amounts of calcification

You are scanning a patient in ICU and notice low-level echoes within the gallbladder consistent with sludge. The gallbladder wall is not thickened. Which statement below is true? a. the patient most likely has acute acalculus cholecystitis b. these findings represent gallbladder perforation c. the patient has sludge most likely due to bile stasis d. the patient has a porcelain gallbladder e. the patient has a pancreatic abnormality

c. the patient has sludge most likely due to bile stasis

The doppler waveform was obtained from the celiac trunk. What technical problem is present? a. the wall filter is set too high b. the wall filet is set too low c. the pulse repetition frequency is set too low d. the pulse repetition frequency is set too hgih e. the baseline is set too low

c. the pulse repetition frequency is set too low

Which of the following statements is NOT true regarding normal anatomy of the kidneys? a. the kidneys are retroperitoneal in location b. the right kidney is located slightly inferior compared to the left c. the tail of the pancreas is in contact with the lateral dorsal aspect of the left kidney d. the superomedial aspect of the right kidney is in contact with the adrenal gland e. the superior pole of each kidney is slightly medial compared to the inferior pole

c. the tail of the pancreas is in contact with the lateral dorsal aspect of the left kidney

You are imaging a patient with suspected benign prostatic hypertrophy (BPH). Which zone is most commonly involved? a. peripheral zone b. central zone c. transition zone d. BPH occurs with equal frequency in all zones e. BPH occurs with equal frequency in the central and transition zone

c. transition zone

Which of the following is NOT a part of the urinary tract? a. kidneys b. ureters c. uterus d. urinary bladder e. urethra

c. uterus

What is the most significant clinical consequence of portal hypertension? a. respiratory compromise b. ascites c. variceal hemorrhage d. liver ischemia e. vascular thrombosis

c. variceal hemorrhage

A 33-year-old male has been referred for a scrotal ultrasound as a part of an infertility work-up. You will tailor your ultrasound exam to include an evaluation for which of the following? a. epididymal cyst b. torsion of the appendix testis c. varicocele d. testicular artery aneurysm e. testicular cyst

c. varicocele

Which anatomic classification is most commonly used to describe the prostate during sonographic evaluation? a. lobar anatomy b. microscopic anatomy c. zonal anatomy d. topographic anatomy e. all of the above are used routinely in ultrasound

c. zonal anatomy

What is the relationship of the splenic vein to the pancreas? a. superior b.anteromedial c.posterior and caudal d. inferior and anterior e. ventral

c.posterior and caudal

What frequency transducer would most commonly be used for endoscopic ultrasound of the pancreas? a. 2.25 MHz b. 3.5 MHz c. 5.0 MHz d. 10 MHz e. 25 MHz

d. 10 MHz

A patient has been referred for a thyroid ultrasound to locate a parathyroid adenoma. How many parathyroid glands are present in most people? a. 1 b. 2 c. 3 d. 4 e. 6

d. 4

You are performing a follow-up study on a patient with a large left renal cyst. Which of the following most accurately describes the prevalence of renal cysts? a. < 1 % of people over age 50 b. 15% of people over age 50 c. 30% of people over age 50 d. 50% of people over age 50 e. 100% people over age 50

d. 50% of people over age 50

You have been asked to provide ultrasound imaging during liver surgery. What transducer would be best suited for this purpose? a. 3.5 MHz curved linear array b. 10 MHz linear array c. 2.25 MHz phased array d. 7 MHz linear array e. 12 MHz curved linear array

d. 7 MHz linear array ( during intraoperative scanning the transducer is placed directly on the exposed liver, for this reason you need a higher frequency, but not too high where you cannot penetrate the whole liver)

You are performin a sonogram on a patient with bilaterally small kidneys. What is the normal range in size for a kidney? a. 2-4 cm b. 4-7 cm c. 7-9 cm d. 9-14 cm e. 13-17 cm

d. 9-14 cm

You are scanning a patient with a history of renal infections. You suspect thinning of the renal cortex. What is the normal measurement of the renal cortex? a. <3mm b. 3-6 mm c. 6-9 mm d. >/= 10 mm e. the renal cortex cannot be measured sonographically

d. >/= 10 mm

You are performing an ultrasound study to rule out the presence of acute appendicitis. Which of the following constitutes an abnormal finding? a. appendix diameter greater than or equal to 6 mm b. noncompressible appendix c. appendix diameter greater than or equal to 3 mm d. A and B only e. B and C only

d. A and B only

A patient has been referred to your ultrasound lab for evaluation of a new renal transplant. Where should you look? a. morison''s pouch b. LUQ c. pouch of douglas d. RLQ e. RUQ

d. RLQ

In performing scrotal sonography, you should tailor your exam to include color doppler analysis in which of the following conditions? a. epididymitis b. torsion c. testicular microlithiasis d. a and b only e. all of the above

d. a and b only

Which of the following would be an indicator of pancreatic transplant rejection? a. high-resistance doppler signals b. low-resistance doppler signals c. heterogeneous parenchyma d. a and c e. b and c

d. a and c

A patient has been referred for ultrasound evaluation with a suspicion of retroperitoneal fibrosis. What structure will you image to identify abnormality? a. pancreas b. kidneys c. adrenal glands d. abdominal aorta e. psoas muscle

d. abdominal aorta (appears as hypoechoic, smoothly marginated, homogeneous masses that involve the abdominal aorta)

This transverse image was obtained in the mid abdomen of a 57- year-old male with lower back pain. Which of the following correctly describes the sonographic findings? a. leaking abdominal aortic aneurysm with hematoma b. abdominal aortic aneurysm and para-aortic lymphadenopathy c. inflammatory abdominal aortic aneurysm d. abdominal aortic aneurysm and horseshoe kidneys e. abdominal aortic aneurysm with retroperitoneal fibrosis

d. abdominal aortic aneurysm and horseshoe kidneys

You are performing a renal sonogram and identify hydronephrosis in the right kidney. Which of the following is NOT a potential cause of hydronephrosis? a. ureteral stone b. large uterine fibroid c. ureteropelvic junction obstruction d. acute pyelonephritis e. ovarian mass

d. acute pyelonephritis

You are scanning a 53-year-old female with a history of recent weight loss and vague abdominal pain. The liver is markedly heterogenous and contains numerous calcified lesions. This most likely represents metastatic disease from which primary? a. non-hodgkin lymphoma b. cystadenocaarcinoma of the ovary c. lung d. adenocarcinoma of the colon e. breast

d. adenocarcinoma of the colon (calcified mets is most commonly associated with mucinous adenocarcinoma of the colon. Mets from the breast, lung, or lymphoma tend to be hypoechoic. Mets from cystadenocarcinoma of the ovary or pancreas are cystic)

A patient is referred for an abdominal Doppler ultrasound to rule out the presence of Budd-Chiari syndrome. What vessels will you evaluate? a. portal vein, splenic vein, renal veins b. hepatic artery, splenic artery, celiac trunk, SMA c. renal veins and renal arteries d. all three hepatic veins, IVC, and the portal vein e. distal abdominal aorta and the iliac arteries

d. all three hepatic veins, IVC, and the portal vein

What is the sonographic appearance of tumefactive sludge within the gallbladder? a. an echogenic mass with prominent color doppler signals b. a mass with low-level echoes with prominent color doppler signals c. a mass containing ringdown artifacts and no color doppler signals d. an avascular mass with low-level echoes e. an adherent, echogenic mass with weak color doppler signals

d. an avascular mass with low-level echoes

A renal mass that is highly echogenic due to its high-fat content is: a. renal cell carcinoma b. wilm's tumor c. renal hamartoma d. angiomyolipoma e. renal lymphoma

d. angiomyolipoma

An ultrasound exam reveals a solid, hyperechoic mass in a 46-year-old patient with tuberous sclerosis. This most likely represents: a. renal cell carcinoma b. Wilm's tumor c. renal hamartoma d. angiomyolipoma e. renal lymphoma

d. angiomyolipoma (50% of patients with tuberous sclerosis will have an angiomyolipoma)

What is the relationship of the left renal vein to the aorta and SMA? a. posterior to both the aorta and SMA b. anteriro to both the aorta and SMA c. parallel and to the left of the aorta and SMA d. anterior to the aorta and posterior to the SMA e. superior to the aorta and inferior to the SMA

d. anterior to the aorta and posterior to the SMA

What part of the pancreas is anterior to the vessel labeled E? a. head b. uncinate process c. neck d. body e. tail

d. body

A patient has been referred to the ultrasound department with a history of medullary nephrocalcinosis. What do you expect to see? a. a calcified renal capsule b. a calcified ureter c. a calcified urinary bladder d. calcified pyramids e. all of the above

d. calcified pyramids

The ligamentum venosum forms the anterior border of what lobe of the liver? a. left lobe b. reidel's lobe c. right lobe d. caudate lobe e. quadrate lobe

d. caudate lobe

The most common benign tumor in the liver is: a. focal nodular hyperplasia b. hepatic adenoma c. hepatic lipoma d. cavernous hemangioma e. hepatoma

d. cavernous hemangioma

The intratesticular arteries that arise from the capsular artery are known as: a. deferential arteries b. cremasteric arteries c. vas deferens d. centripetal arteries e. pampiniform plexus

d. centripetal arteries

Which of the following arteries courses within the testicular parenchyma? a. testicular artery b. deferential artery c. cremasteric artery d. centripetal artery e. all of the above

d. centripetal artery

Regenerating nodules are a feature associated with: a. hepatitis b. hepatocellular carcinoma c. hydatid disease d. cirrhosis e. polycystic liver disease

d. cirrhosis

You are imaging a patient with dilated loops of bowel. you detect the presence of the haustra. This is an identifying feature of which part of the GI tract? a. stomach b. duodenum c. small intestine d. colon e. all of the above

d. colon

You have been asked to evaluate a suspicious gut mass with color and spectral Doppler. What is the role of doppler in the evaluation of gut masses? a. color doppler is definitice in determination if a mass is benign or malignant b. color doppler is used to identify regions of peristalsis c. color doppler is useful to identify the layers of the gut d. color doppler aids in differentiation between ischemic and inflammatory gut masses e. there is no role for color or spectral doppler in the evaluation of the GI tract at this time

d. color doppler aids in differentiation between ischemic and inflammatory gut masses (ischemic will not have color flow, inflammatory issues will)

You are reviewing lab work prior to performing an abdominal ultrasound exam. Elevated lab values include Gamma-glutamyl transpeptidase (GGT) and alkaline phosphatase. Which statement below is true? a. elevation of both GGT and ALK phos suggests the source of elevated ALK phos is due to metastatic bone cancer b. elecation of both GGT and ALK phos is a sensitive indicator of pancreatitis c. If both GGT and ALK phos are elevated, the lab work is invalid and must be repeated d. concomitant elevation of both GGT and ALK phos indicates the source of the elevated ALK phos in the liver e. concomitant elevation of both of these lab values is highly specific for hepatocellular carcinoma

d. concomitant elevation of both GGT and ALK phos indicates the source of the elevated ALK phos in the liver (ALK phos can be elevated for a ton of things but if GGT is also elevated it means the ALK phos is raised by something in the liver)

You are using color doppler to evaluate for portal cein patenct. The frame rate is very slow. What parameter can you adjust to increase the color doppler frame rate? a. increase the color filter b. decrease the color pulse repetition frequency c. increase the color gain d. decrease the color field of view e. increase the color baseline

d. decrease the color field of view

You have obtained this sagittal image in a 27-year-old female with epigastric pain. The small arrows are pointing to what structure? a. esophageal gastricjunction b. left adrenal gland c. right adrenal gland d. diaphragmatic e. left renal vein

d. diaphragmatic crura

You are performing a thyroid ultrasound on a patient with a history of Hashimoto's thyroiditis. What is the sonographic appearance of this condition? a. multiple large, hypoechoic modules throughout both lobes b. many small, echogenic nodules throughout both lobes c. small, shrunken thyroid with heterogeneous echo texture d. diffuse enlargement of the thyroid with heterogeneous echo texture e. enlarged thyroid with homogeneous low level echo texture

d. diffuse enlargement of the thyroid with heterogeneous echo texture

You are scanning at the area of the porta hepatis in a patient with alcoholic liver cirrhosis. Two large tubular structures are identified. How can you identify which structure is the duct and which is the hepatic artery? a. the hepatic artery is always located between the portal vein and the bile duct b. the bile duct can be compressed with probe pressure and the hepatic artery is not easily compressed c. the bile duct will dilate with a valsalva maneuver and the hepatic artery will not dilate d. doppler signals can be elicited from the artery but not the bile duct e. all of the above

d. doppler signals can be elicited from the artery but not the bile duct

You are performing a prostate ultrasound in a patient having an infertility work-up. Which cyst may be associated with infertility? a. mullerian duct cyst b. prostatic utricle cyst c. retention cyst d. ejaculatory duct cyst e. seminal vesicle cysts

d. ejaculatory duct cyst

What is the purpose of obtaining images in the left lateral decubitus position during ultrasounography of the gallbladder? a. eliminate the presence of gas in the image of the gallbladder b. rotate the CBD so that it is imaged over the portal vein c. allow visualization of the cystic duct d. evaluate mobility of gallstones e. see if the patient has pain when lying on the left side

d. evaluate mobility of gallstones

You have a patient scheduled for a gallbladder sonogram. What preparation is required? a. there is no necessary prep b. patient should drink 4-6 8 oz glasses of water to improve hydration prior to the study c. eat a fatty meal 30 minutes prior to study d. fasting for 8-12 hours prior to study e. fasting at least 24 hours prior to study and ingest an anti-gas medication

d. fasting for 8-12 hours prior to study

You are scanning a patient with autosomal dominant polycystic kidney disease. Which of the following statements is NOT true regarding this disease? a. liver cysts may be present in up to 30% of patients b. high blood pressure is common c. cysts may be complicated by bleeding or infection d. frequently only one kidney is involved e. progressive renal failure is common

d. frequently only one kidney is involved

You are asked to perform a follow-up study on a patient with a known abdominal aortic aneurysm. Which of the following terms correctly describes this aneurysm? a. pseudoaneurysm b. saccular c. dissecting d. fusiform e. dumbbell

d. fusiform

You are scanning a patient with a porcelain gallbladder. You must carefully evaluate the gallbladder because these patients are at increased risk for: a. adenomyomatosis b. cholesterolosis c. choledocholithiasis d. gallbladder carcinoma e. gallbladder perforation

d. gallbladder carcinoma

You have obtained a sagittal image of the pancreatic head and detect a small, tubular structure coursing cephalocaudad anterior to the pancreas. What is this structure? a. common bile duct b. superior mesenteric artery c. common hepatic artery d. gastroduodenal artery e. left gastric artery

d. gastroduodenal artery

You are imaging a patient presenting with trauma, pain, and scrotal swelling. You detect a large hydrocele with prominent internal echoes. This most likely represents: a. pyocele b. simple hydrocele c. varicocele d. hematocele e. hernia

d. hematocele

A patient is referred with RUQ tenderness and a history of oral contraceptive use. A solid, hypoechoic mass is identifies in the right lobe of the liver. Color doppler reveals hypervascularity of the mass. Which of the following scenarios is most likely? a. hydatid liver disease b. hepatic lipoma c. hepatic abscess d. hepatic adenoma e. hepatocellular carcinoma

d. hepatic adenoma

A 49-year-old male is referred for abdominal doppler with a history of cirrhosis and portal hypertension. Ultrasound findings related to portal hypertension include all of the following except: a. splenomegaly b. portosystemic varices and collaterals c. hepatofugal flow in the portal vein d. hepatic cysts e. dilated portal vein

d. hepatic cysts

You have obtained this waveform from an abdominal vessel. What vessel are you sampling? a. SMV b. portal vein c. renal vein d. hepatic vein e. splenic vein

d. hepatic vein

You are scanning a patient with a history of fever, abnormal liver function tests, and RUQ tenderness. The liver is enlarged with decreased echogenicity, the GB wall is thickened and thick echogenic bands are noted surrounding the portal veins. Which of the following conditions is most likely? a. fatty liver b. cirrhosis c. budd-chiari syndrome d. hepatitis e. normal liver

d. hepatitis

You are performing a doppler study of the mesenteric arteries in a fasting patient. Which of the following describes the waveform you expect to see in a normal study? a. low resistance b. retrograde c. continuous d. high resistance e. monophasic

d. high resistance

You are performing an ultrasound study on a patient with an aortoiliac graft and suspect the presence of a pseudoaneurysm at the graft site. What is the typical waveform in the neck of a pseudoaneurysm? a. continuous b. low velocity, monophasic c. tardus parvus d. high velocity, bidirectional e. high velocity, low resistance

d. high velocity, bidirectional

Which of the following is a classic sonographic feature of Crohn's disease? a. gut wall thickening b. strictures c. creeping fat d. increased vascularity e. all of the above

e. all of the above

A patient has been referred from CT with a history of nephrocalcinosis. What is the ultrasound appearance of this entity? a. normal sized kidney with focal, wedge shapped hypoechoic mass b. multiple hypoechoic masses throughout the kidney c. echogenic kidney with calcified capsule d. highly echogenic renal pyramids with or without posterior acoustic shadowing e. cystic masses containing tiny echogenic foci situated throughtout the kidney

d. highly echogenic renal pyramids with or without posterior acoustic shadowing

You have been asked to identify a parathyroid adenoma. What is the typical sonographic appearance of this structure? a. hyperechoic, round, solid nodule b. hetergeneous, oval, solid nodule c. round, solid nodule, isoechoic to the thyroid gland d. homogeneous, hypoechoic, solid oval-shaped nodule e. hypoechoic nodule with highly echogenic hilum

d. homogeneous, hypoechoic, solid oval-shaped nodule

What term best describes this lower pole renal mass? a. hypoechoic b. anechoic c. isoechoic d. hyperechoic e. none of the above

d. hyperechoic

Where should you look for the pancreas in a patient with a pancreatic transplant? a. pouch of douglas b. morison's pouch c. LUQ d. iliac fossa e. epigastrium

d. iliac fossa

Which of the following is NOT a feature of hepatic cysts? a. thin wall b. posterior acoustic enhancement c. anechoic d. increased attenuation e. increased through transmission

d. increased attenuation

Which of the following is NOT a feature you would detect in a spenic cyst? a. smooth border b. posterior acoustic enhancement c. anechoic d. increased attenuation e. rounded shape

d. increased attenuation

A patient is referred to rule out hepatomegaly. All of the following are useful indicators of hepatomegaly EXCEPT: a. rounding of the ingerior border of the liver b. longitudinal measurement of the right lobe exceeding 15.5 cm c. extension of the right lobe inferior to the lower pole of the right kidney d. increased diameter of the main portal vein greater than 1 cm e. increased anteroposterior measurement of the right lobe

d. increased diameter of the main portal vein greater than 1 cm (enlargement of the portal vein is a finding associated with portal hypertension but is not a measurement of hepatomegaly)

You have been asked to evaluate the tail of the pancreas. What is the relationship of the pancreatic tail to the spleen? a. superior and lateral b. superior and medial c. anterior d. inferomedial e. posterior

d. inferomedial

The best way to identify the intrahepatic biliary system is the image which structures below? a. hepatic veins b. all fissures and ligaments within the liver parenchyma c. intrahepatic lymphatics d. intrahepatic portal veins e. the intrahepatic biliary system cannot be detected sonographically

d. intrahepatic portal veins

You have been asked to perform an abdominal ultrasound on a patient with abnormal blood urea nitrogen (BUN) levels. All other lab work is normal. Considering this history, what is the study area of interest? a. pancreas b. liver c. adrenal d. kidney e. aorta

d. kidney

Identify the vessels labeled "A" and "B" a. common hepatic artery and superior mesenteric artery b. proper hepatic artery and celiac trunk c. right gastric artery and celiac trunk d. left gastric artery and celiac trunk e. proper hepatic artery and splenic artery

d. left gastric artery and celiac trunk

You are performing an ultrasound exam on an adolescent male to rule out testicular torsion. How should you adjust the system color doppler parameters to achieve the greatest sensitivity to slow flow? a. high PRF, low filter, high gain, high packet size b. low PRF, low filter, high gain, low packet size c. high PRF, high filter, high gain, high packet size d. low PRF, low filter, high gain, high packet size e. low PRF, low filter, low gain, low packet size

d. low PRF, low filter, high gain, high packet size ( increasigng packet size increases the number of sound pulses per line)

You are performing a doppler evaluation of the testis. Which of the following describes the waveform you will see within an artery of the testicular parenchyma if the testis is normal? a. high resistance b. bidirectional c. phasic d. low resistance e. continuous

d. low resistance ( the testicular artery = low resistance, cremasteric and deferential arteries = high resistance)

Apatient has been referred to your lab for a Doppler study of the kidneys. What type of waveform do you expect to see in the normal main renal artery? a. high resistance with prominent systolic flow and little diastolic flow b. continuous with little differentiation between systole and diastole c. high impedance with no diastolic component d. low resistance with forward flow throughout the cardiac cycle e. prominent early systolic peak with retrograde flow in early diastole

d. low resistance with forward flow throughout the cardiac cycle

You are scanning the gallbladder and notice that the wall is abnormally thickened. Which of the following is NOT a cause of gallbladder wall thickening? a. inflammation b. hepatic dysfunction c. congestive heart failure d. malignant ascites e. gallbladder wall varices

d. malignant ascites (usually associated with a normal gallbladder wall diameter)

You are perfoming an ultrasound study on a patient with malignant melanoma. Your ultrasound findings reveal multiple hyperechoic masses within the spleen. This most likely represents: a. histoplasmosis b. tuberculosis c. pseudocyst d. metastasis e. infarction

d. metastasis

The most common form of malignant disease of the liver is? a. hepatocellular carcinoma b. angiosarcoma c. cholangiocarcinoma d. metastatic disease e. primary lymphoma

d. metastatic disease (hepatocellular carcinoma is the most common PRIMARY malignant neoplasm, but metastatic disease is by far the most common form of malignancy in the liver)

You are scanning a patient with a known mass in the left medial segment of the liver. What anatomic landmark can you use to identify the left medial segment separate from the right anterior segment of the liver? a. left portal vein b. ligamentum teres c. ligamentum venosum d. middle hepatic vein e. left hepatic vein

d. middle hepatic vein

What preperation should you require of your patient scheduled for renal sonograms? a. fasting for 24 hours prior to examination b. ingestion of 100 mg simethicome 5 minutes before examination c. water enema d. moderate hydration with no other specific preparation e. faty meal within 30 minutes of examination

d. moderate hydration with no other specific preparation

You are imaging the spleen of a 27-year-old male with AIDS. What is the most common finding in the spleen in patients with AIDS? a. abscess b. metastasis c. accessory spleen d. moderate splenomegaly e. lymphoma

d. moderate splenomegaly (most common in moderate splenomegaly, also involved with focal lesions like candida, pneumocystis, or mycobacterium. Also may be involved in Kaposi's sarcoma or lymphoma)

WHat is the purpose of a quality control program in the ultrasound department? a. evaluate sonographer scanning time b. reduce patient exposure c. compare exam fees with competitors d. monitor equipment performance e. all of the above

d. monitor equipment performance

You are performing an abdominal ultrasound on a patient with histoplasmosis. What findings of the spleen are commonly seen with this condition? a. small, shrunken spleen b. multiple hypoechoic masses throughout the spleen c. splenic abscess d. multiple focal, bright echogenic granulomatous lesions throughout the spleen e. solitary cyst with calcified walls

d. multiple focal, bright echogenic granulomatous lesions throughout the spleen

A patient is referred for ultrasound evaluation of possible cavernous transformation of the portal vein. Which of the following describes this condition? a. retrograde portal venous flow b. portal vein thrombosis c. tumor invasion of the portal vein d. network of vessels replacing obliterated portal vein e. portal vein aneurysm

d. network of vessels replacing obliterated portal vein

A large complex hydrocele is most commonly associated with which of the following? a. seminoma b. epidiymal cyst c. varicocele d. orchitis e. infarction

d. orchitis

You are performing a thyroid ultrasound on a patient with increased serum calcium levels. What pathology are you searching for? a. hashimoto's thyroiditis b. grave's disease c. adenomatois thyroid nodule d. parathyroid adenoma e. colloid cyst

d. parathyroid adenoma

A patient has been referred for doppler interrogation of the kidney. Which view provides the best color doppler evaluation of the intrarenal vasculature? a. patient supine, anterior view through liver b. patient supine, coronal view through liver c. patient prone, oblique view through back musculature d. patient in posterior oblique position, coronal view through posterior axillary line e. patient upright, anterior view through liver

d. patient in posterior oblique position, coronal view through posterior axillary line

You are performing a transrrectal prostate ultrasound in a sagittal plane. The most lateral images of the gland show which tissue? a. transition zone b. central zone c. periurethral area d. peripheral zone e. fibromuscular stroma

d. peripheral zone

You are performing and ultrasound on a patient with TIPS. What 2 vessels are connected with the TIPS stent? a. portal vein and hepatic artery b. hepatic artery and hepatic vein c. hepatic vein and IVC d. portal vein and hepatic vein e. portal vein and IVC

d. poral vein and hepatic vein

You obtained this doppler waveform while performing an abdominal doppler ultrasound study. Which of the vessels below were you most likely sampling? a. hepatic vein b. hepatic artery c. IVC d. portal vein e. aorta

d. portal vein

Oxygenated blood is supplied to the liver via the: a. portal vein and hepatic vein b. hepatic vein and hepatic artery c. hepatic vein and portal vein d. portal vein and hepatic artery e. hepatic artery only

d. portal vein and hepatic artery

You have been asked to identify the seminal vesicles. What is their anatomic relation to the prostate gland? a. posterior and inferior b. anterior and inferior c. anterior and superior d. posterior and superior e. medial

d. posterior and superior

Vessel A is located in what lobe of the liver? a. caudate lobe b. medial segment left lobe c. lateral segment left lobe d. posterior segment right lobe e. anterior segment right lobe

d. posterior segment right lobe

Which of the following descrives the normal course of the right renal artery? a. retroaortic b. between the sma and aorta c. anterior to sma and ivc d. posterior to ivc e. between sma and splenic vein

d. posterior to ivc

You have been asked to identify the right adrenal gland. Which of the following correctly describes its location? a. medial to the inferior vena cava b. superolateral to the upper pole of the right kidney c. between the inferior vena cava and the abdominal aorta d. posterior to the inferior vena cava e. medial to the crus of the diaphragm

d. posterior to the inferior vena cava (located superomedial to the upper pole of the right kidney and posterior to the IVC, lateral to the crus of the diaphragm)

You are imaging the common hepatic artery and detect its division into two branches. What are these two branches? a. proper hepatic artery and right gastric artery b. right gastric artery and gastroduodenal artery c. left gastric artery and proper hepatic artery d. proper hepatic artery and gastroduodenal artery e. gastroduodenal artery and left gastric artery

d. proper hepatic artery and gastroduodenal artery

You have been asked to perform a transrectal ultrasound of the prostate in a patient with metastasis of unknown origin. WHat are you looking for? a. bening prostatic hyperplasia b. ejaculatory duct cysts c. prostatic abscess d. prostate cancer e. prostatitis

d. prostate cancer

In the image below, what structure does the letter "B" represent? a. rib b. lumbar spine c. quadratus lumborum muscle d. psoas muscle e. obturator internus muscle

d. psoas muscle

Which of the following results from an ascending urinary tract infection? a. horseshoe kidney b. acute tubular necrosis c. glomerulonephritis d. pyelonephritis e. nephrocalcinosis

d. pyelonephritis

Which structure would you image posterior to the prostate gland? a. urinary bladder b. pubic bones c. levator ani muscles d. rectum e. urethra

d. rectum

A patient is referred for ultrasound evaluation to rule out the presence of renal malignancy. What is the most common solid renal mass in the adult? a. oncocytoma b. transitional cell carcinoma c. angiomyolipoma d. renal cell carcinoma e. adenoma

d. renal cell carcinoma

You are performing a follow-up sonogram on a patient in which a 5-mm cyst was previously identified at the anterior border of the left lobe of the liver. Although you are using a3.5 MHz curved linear array probe, you do not see the cyst. Which of the following would be most helpful in improving visibility of this cyst? a. increase the overall gain b. increase the dynamic range c. increase the transmit power d. rescan the left lobe with a higher frequency transducer e. rescan the left lobe with a lower frequency linear array transducer

d. rescan the left lobe with a higher frequency transducer

Which retroperitoneal compartment contains the psoas and quadratus lumborum muscles? a. anterior pararenal space b. parirenal space c. posterior pararenal space d. retrofascial space e. none of the above

d. retrofascial space

What is the etiology of the low-level echoes seen in the near field of this gallbladder? a. biliary sludge b. tumefactive sludge c. floating cholesterol stones d. reverberation artifact e. gas due to emphysematous cholecystitis

d. reverberation artifact

A patient has been referred to your ultrasound lab for evaluation of a new renal transplant. Where should you look? a. Morison's pouch b. left upper quadrant c. pouch of douglas d. right lower quadrant e. right upper quadrant

d. right lower quadrant

You have obtained this image of the urinary bladder ina patient with a suspected right ureteral obstruction. What is represented on this image? a. transitional cell carcinoma involving the bladder b. endometriosis of the urinary bladder c. left ureteral jet d. right ureteral jet e. foley catheter within the urinary bladder

d. right ureteral jet

A 51-year-old male is referred for abdominal ultrasound with abnormal liver function tests and jaundice. Which lab work would aid in differentiation of an intrahepatic versus extrahepatic cause of jaundice? a. serum alpha-fetoprotein b. alkaline phosphatase c. Aspartate aminotransferase d. serum bilirubin e. serum creatinine

d. serum bilirubin

A patient presents to the ultrasound department for a sonogram to rule out biliary obstruction. Which lab test would best indicate the presence of bile duct obstruction? a. serum creatinine b. serum amylase c. serum lipase d. serum direct bilirubin e. alphafetoprotein

d. serum direct bilirubin

A patient has been referred for sonographic evaluation of the rotator cuff. What part of the body should you evaluate? a. hip b. elbow c. wrist d. shoulder e. ankle

d. shoulder

You have been asked to include an evaluation of the SMV in a doppler study of the portal system. What vessel does the SMV join? a. IMV b. left renal vein c. right renal vein d. splenic vein e. hepatic vein

d. splenic vein

Which of the following does NOt drain directly into the IVC? a. renal vein b. right hepatic vein c. common iliac vein d. splenic vein e. middle hepatic vein

d. splenic vein (splenic vein joins the SMV to form the portal vein)

You have identified the thyrocervical trunk in a patient with a hypervascular thyroid nodule. The thyrocervical trunk arises from what artery? a. common carotid artery b. internal carotid artery c. external carotid artery d. subclavian artery e. vertebral artery

d. subclavian artery

What part of the pancreas is NOT demonstrated in this image? a. head b. neck c. body d. tail e. a and b

d. tail

The capsular artery of the testis is a branch of what artery? a. deferential b. abdominal c. cremasteric d. testicular e. internal iliac

d. testicular

During a routine renal ultrasound, you are suspicious of increased echogenicity of the kidneys. Which of the following describes the normal echogenicity of the renal cortex? a. the kidney is normally hyperechoic in comparison to the spleen and liver b. the kidney echogenicity is always hypoechoic compared to the spleen and liver c. the normal kidney is never isoechoic with the liver d. the normal kidney echogenicity is frequently isoechoic with the liver and spleen e. the echogenicity of the kidney

d. the normal kidney echogenicity is frequently isoechoic with the liver and spleen

You are asked to perform a doppler study of the hepatic veins in the liver. What differentiates the hepatic veins from the portal veins? a. the hepatic veins converge toward the porta hepatis b. the hepatic veins have brightly echogenic walls c. the portal veins are largest near the dome of the liver d. the portal veins are accompanied by branches of the biliary tree and hepatic artery e. the portal veins normal exhibit a triphasic flow pattern

d. the portal veins are accompanied by branches of the biliary tree and hepatic artery

You are reviewing a CT report on a patient referred for abdominal sonography. The report states that pancreatic divisum is present. What does this mean? a. the pancreas is split into two pieces on each side of the abdomen b. the pancreas head is separate from the body and tail c. the pancreatic duct is duplicated d. the two pancreatic ducts have not fused e. the pancreas is malrotated

d. the two pancreatic ducts have not fused

You detect irregular thickening of the bladder wall in a 53-year old male with hydronephrosis and a dilated ureter. Which of the following would you suspect? a. renal cell carcinoma b. ureterocele c. bladder outlet obstruction d. transitional cell carcinoma e. endometriosis

d. transitional cell carcinoma

You have been asked to perform an ultrasound study on a patient with a pancreatic transplant. What chronic condition does this patient probably have? a. crohn's disease b. chronic pancreatitis c. lymphoma d. type 1 diabetes mellitus e. lupus

d. type 1 diabetes mellitus (performed to decrease insulin dependency)

What is the indication for a Doppler renal study to rule out renal artery stenosis? a. hematuria b. increased serum creatinine c. leukocytosis and fever d. uncontrolled hypertension e. anemia, progressive azotemia and polyuria

d. uncontrolled hypertension

A 57-year-old woman has been referred for abdominal ultrasound with a history of an abdominal bruit. What pathology are you searching for? a. cholecystitis b. para-aortic lymphadenopathy c. retroperitoneal fibrosis d. vascular compromise involving the aorta or its branches e. varices

d. vascular compromise involving the aorta or its branches

Which transducer will provide the most optimal imaging for ultrasound imaging of the breast? a. 3.5 MHz curved linear array b. 5.0 MHz curved linear array c. 5.0 MHz linear array d. 7.5 MHz linear array e. 10.0 MHz linear array

e. 10.0 MHz linear array (for a large patient you may need 7.5 MHz but overall 10.0 MHz is the best option)

You are imaging the gut with a high-resolution trasducer. How many layers do you expect to see in a good image of the gut? a. 1 b. 2 c. 3 d. 4 e. 5

e. 5 (1. interface of superficial mucosa with luminal content 2. deep mucosa 3. submucosa 4. muscularis propria 5. serosa)

You have been asked to perform a transrectal prostate ultrasound exam. What frequency transducer is best suited for this study? a. 2-3 MHz b. 3-4 MHz c. 4-5 MHz d. 5-6 MHz e. 7-8 MHz

e. 7-8 MHz

You suspect intrahepatic bile duct dilatation in a patient with RUQ pain and tenderness. How can you differentiate the dilated bile ducts from intrahepatic veins? a. dilated bile ducts demonstrate irregular tortuous walls b. intrahepatic portal veins show increased through transmission compared to the dilated bile ducts c. bile ducts will not demonstrate flow with color doppler d. all of the above e. A and C only

e. A and C only

Which retroperitoneal compartment contains no solid organs? a. anterior pararenal space b. parirenal space c. posterior pararenal space d. retrofascial space e. C and D

e. C and D

Identify the anatomy labeled "C" a. aorta b. portal vein c. superior mesenteric vein d. inferior mesenteric vein e. Inferior vena cava

e. Inferior vena cava

What ligament divides the left lobe of the liver into medial and lateral segments? a. ligamentum venosum b. hepatoduodenal ligament c. main lobar fissure d. coronary ligament e. ligamentum teres

e. Ligamentum teres

You are imaging a patient with abdominal pain and detect multiple dilated fluid-filled loops of bowel. This is suspicious for which of the following? a. obstruction b. ileus c. lymphoma d. appendicitis e. a and b

e. a and b

You have just discovered a pancreatic mass suspicious for adenocarcinoma in a patient with weight loss and abdominal pain. What associated findings should you look for? a. lymphadenopathy b. liver metastasis c. portal vein aneurysm d. aortic aneurysm e. a and b

e. a and b

You detect a mass within the left testis on a 43-year-old male. Which of the following is a sign indicative of malignancy? a. irregular shape of testis b. intratesticular location of mass c. presence of a large hydrocele d. all of the above e. a and b only

e. a and b only

You have detected an exophytic mass attached to the gut and suspect malignancy. You should tailor your exam to specifically evaluate what related pathology? a. regional lymph node enlargement b. liver metastasis c. pancreatic adenocarcinoma d. pancreatitis e. a and b only

e. a and b only

You are scanning a patient with moderate splenomegaly. Which of the following is a cause of mild to moderate splenomegaly? a. portal hypertension b. infection c. AIDS d. lymphoma e. a, b, and c only

e. a, b, and c only (lymphoma or leukemia would result in marked splenomegaly)

You are performing an abdominal ultrasound on a patient with a history of lung cancer. What retroperitoneal structure is a common site for metastasis from the lung? a. pancreas b. kidneys c. psoas muscle d. duodenum e. adrenal glands

e. adrenal glands

You are performing an abdominal ultrasound on a patient with a history of pheochromocytoma. Sonographic evaluation of what structure will be most beneficial in identification of this abnormality? a. pancreas b. kidneys c. psoas muscle d. abdominal aorta e. adrenal glands

e. adrenal glands

A patient is regerred for a follow-up abdominal sonogram. The previous report indicates that pneumobilia was present. What is pneumobilia? a. perforation of the bile ducts b. biliary duct dilatation c. common bile duct stones d. air in the gallbladder e. air in the bile ducts

e. air in the bile ducts

A questionable mass is seen between the renal pyramids on the right kidney. You suspect this is a Column of Bertin "pseudomass". Which of the following sonographic features helps distinguish this from a true pathologic mass? a. isoechogenicity with the rest of the renal cortex b. continuity with the renal cortex c. lack of mass effect or splaying of central renal sinus fat d. normal vascularity by color doppler e. all of the above

e. all of the above

During ultrasound evaluation of the gallbladder system, you notice thickening of the bile duct walls. This finding may be related to which of the following? a. sclerosing cholangitis b. pancreatitis c. choledocholithiasis d. cholangiocarcinoma e. all of the above

e. all of the above

Following liver transplantation, which of the following anatomic locations has an anastomotic connection that should be evaluated with ultrasound? a. inferior vena cava b. portal vein c. hepatic artery d. bile duct e. all of the above

e. all of the above

The tail of the pancreas is in contact with which of the following structures? a. left kidney b. splenic flexure of the colon c. spleen d. a and c e. all of the above

e. all of the above

This view is useful in the evaluation of what abnormality? a. renal artery stenosis b. multiple renal arteries c. aortic aneurysm d. IVC thrombosis e. all of the above

e. all of the above

Ultrasound imaging reveals left-sided hydronephrosis in a 38-year-old woman with vague abdominal pain. You should tailor your exam to rule out which of the following? a. ureteral calculi b. pelvic mass c. aortic aneurysm d. a and b e. all of the above

e. all of the above

WHich of the following is suggestive of malignancy of a solid mass seen on ultrasound? a. a mass that is taller than it is wide b. spiculation c. angular margins d. markedly hypoechoic solid lesion e. all of the above

e. all of the above

Which of the following describes an abnormal sonographic appearance of the gut? a. target b. asymmetric target c. pseudokidney d. a and b only e. all of the above

e. all of the above

Which of the following is a pitfall you might encounter in ultrasound scanning of the spleen? a. intercostal scanning may produce rib shadows obscuring the splenic parenchyma b. in hepatomegaly, the left lobe of the liver may be mistaken for the spleen c. a mass in the tail of the pancreas may be confused with a splenic mass d. an accessory spleen may be confused with an enlarged lymph node e. all of the above

e. all of the above

Which of the following is a risk factor for the development of pancreatic cancer? a. smoking b. high fat diet c. diabetes d. chronic pancreatitis e. all of the above

e. all of the above

Which of the following is a symptom associated with acute cholecystitis? a. nausea b. vomiting c. epigastric pain d. RUQ pain e. all of the above

e. all of the above

Which of the following is indication for transrectal ultrasound of the prostate? a. abnormal digital rectal exam b. abnormal laboratory test results indicative of prostate cancer c. guidance for directed sonographic biopsy d. monitor response to treatment of prostate cancer e. all of the above

e. all of the above

Which of the following parameters can be tested on a phantom? a. sensitivity b. contrast resolution c. axial resolution d. lateral resolution e. all of the above

e. all of the above

While performing a thyroid ultrasound, you detect a lymph node suspicious for malignancy. WHich of the following sonographic features is consistent with nodal malignancy in the neck? a. rounded lymph node b. heterogeeous echo texture c. intranodal calcification d. absence of the echogenic hilum e. all of the above

e. all of the above

You ahve been asked to evaluate an atypical renal cyst seen on CT. What feature below is indicative of an atypical cyst? a. internal septations b. wall calcification c. internal echoes d. irregular walls e. all of the above

e. all of the above

You are performing Doppler analysis of the spermatic cord. Which of the following arteries or structures is found within the spermatic cord? a. vas deferens b. testicular artery c. cremasteric artery d. deferential artery e. all of the above

e. all of the above

You are performing an abdominal ultrasound and are having difficulty identifying the spleen. Which of the following structures is in contact with the spleen? a. left hemidiaphragm b. stomach c. pancreas d. splenic flexure of the colon e. all of the above

e. all of the above

You are performing an abdominal ultrasound and suspect a left adrenal mass. Which of the following conditions may simulate an adrenal mass? a. thickened diaphragmatic crus b. accessory spleen c. gastric diverticulum d. retroperitoneal lymphadenopathy e. all of the above

e. all of the above

You are performing an abdominal ultrasound to search for lymphadenopathy. Where should you look? a. splenic hilum b. porta hepatis c. renal hilum d. para-aortic area e. all of the above

e. all of the above

You are requested to perform an abdominal ultrasound on a patient to evaluate for complications of pancreatitis. What should you look for? a. pseudoaneurysm b. pseudocyst c. phlegmon d. abscess e. all of the above

e. all of the above

You are scanning a patient with sickle cell anemia and note the presence of gallstones and gallbladder wall thickening. What else should you do to determine if acute cholecystitis is present? a. press with the ultrasound probe over the gallbladder to determine if its painful b. look carefully to see if a gallstone is lodged in the gallbladder neck c. check for the presence of pericholecystic fluid d. A and B only e. all of the above

e. all of the above

You are scanning a patient with symtoms of cholelithiasis. Although you cannot clearly identify a gallbladder, you detect a bright band of echoes with posterior shadowing in the right upper quadrant. How can you determine if this represents a contracted gallbladder filled with stones? a. connection of the shadowing echoes to the interlobar fissure confirms identification of the gallbladder b. the wall-echo-shadow sign confirms identification of the gallbladder c. "Dirty" shadowing from bowel fas can be differentiated from "clean" shadowing from stones by the presence of ringdown artifact in the bowel gas shadow d. a and c only e. all of the above

e. all of the above

You are scanning the gallbladder and notice some smudgy echoes within it. You suspect the echoes are due to artifact. What is a common cause of artifactual echoes within the gallbladder? a. reverberation b. side lobes c. slice thickness artifact d. a and b only e. all of the above

e. all of the above

You have been asked to perform an ultrasound to evaluate for biliary obstruction in a pateint with a history of weight loss and midepigastric pain. You find both intrahepatic and extrahepatic biliary dilatation. The gallbladder is hydropic. Which of the following conditions cauing ductal dilatation should you look for? a. choledocholithiasis b. pancreatic carcinoma c. chronic pancreatitis with stricture formation d. a and b only e. all of the above

e. all of the above

You have been asked to rule out pseudocyst formation in a patient with acute pancreatitis. What is the ultrasound appearance of a pancreatic pseudocyst? a. cyst without internal echoes b. cyst with low-level echoes c. cyst with internal septations d. well-defined wall e. all of the above

e. all of the above

You have detected a solid mass in the right kidney of a 47-year-old male. You should tailor your exam to evaluate which of the following? a. extension of tumor into the renal vein b. search for liver metastasis c. search for retroperitoneal adenopathy d. a and b only e. all of the above

e. all of the above

You have detected a transtesticular artery during scrotal sonography. Which of the following is TRUE regarding this finding? a. it is a common anatomic variant b. it courses in the opposite direction of the centripetal arteries c. it enters the testicular parenchyma at the mediastinum testis d. a large vein frequently accompanies it e. all of the above

e. all of the above

You have detected an abdominal aortic aneurysm. Considering this finding, you should tailor your exam to include which of the following? a. evaluation of the common iliac arteries b. measurement of the transverse and anteroposterior diameter c. assessment of intraluminal thrombus d. location of the aneurysm in relation to the renal arteries e. all of the above

e. all of the above

You have documented the presence of a pseudocyst adjacent to the pancreatic head in a 56-year-old male. Pseudocyst may be associated with which of the following? a. acute pancreatitis b. chronic pancreatitis c. pancreatic cancer d. a and b only e. all of the above

e. all of the above

You suspect hydronephrosis in a 42-year-old female who complains of vague abdominal discomfort. Which of the following is a cause of false-positive determination of hydronephrosis? a. overdistention of the urinary bladder b. parapelvic cysts c. prominent hilar vessels d. large extrarenal pelvis e. all of the above

e. all of the above

You have detected compensatory hypertrophy of the right kidney in a 35-year-old male. This finding is associated with which of the following? a. nephrectomy b. renal agenesis c. renal hypoplasia d. renal atrophy e. all of the above

e. all of the above (compensatory hypertrophy is enlargement of the kidney. Seen with all of these things because the one kidney is working extra hard to compensate for the absence of work from the contralateral kidney)

What is the sonographic appearance of prostate cancer? a. hypoechoic lesions b. isoechoic lesions c. hyperechoic lesions d. only a and b e. all of the above appearances have been encountered

e. all of the above appearances have been encountered

A patient is referred for a sonogram of the liver to rule out metastatic disease. Which of the following describes the sonographic appearance of liver metastasis? a. single hypoechoic mass b. multiple hyperechoic masses c. masses of mixed echogenicity d. cystic masses e. all of the above appearances of liver metastasis may be encountered

e. all of the above appearances of liver metastasis may be encountered

You are imaging the abdomen in a patient with abdominal pain and weight loss. You detect an area that is suspicious for mass, but are considering the possibility that the mass is normal loop of bowel. What should you do? a. have the patient return for a comparison study b. image the area for a short duration to see if peristalsis occurs c. scan the patient in a different position d. evaluate the area with color doppler e. all of the above maneuvers should be attempted if diagnostic confidence is not attained

e. all of the above maneuvers should be attempted if diagnostic confidence is not attained

Focal fatty liver is most commonly found in which location? a. medial to the ascending branch of the left portal vein b. posterior to the right hepatic vein c. lateral, inferior rip of the right lobe d. adjacent to the fissure for the ligamentum venosum e. anterior to the portal vein at the porta hepatis

e. anterior to the portal vein at the porta hepatis

A patient is referred for ultrasound with a history of liver transplantation. You identify an extrahepatic fluid collection. What is the likely etiology of this finding? a. biloma b. hematoma c. loculated ascites d. abscess e. any of the above may be seen following liver transplantation

e. any of the above may be seen following liver transplantation

On the left side of the neck the common carotid arises from what artery? a. innominate b. subclavian c. right common carotid d. internal jugular e. aortic arch

e. aortic arch

You are aiding a physician in a transrectal biopsy of the prostate. What patient preparation is required? a. no patient preparation is required for a transrectal biopsy b. a clensing enema is performed immediately prior to the procedure c. antibiotics are administered prior to and following the examination d. the patient is requested to have nothing to eat or drink 12 hours prior to the procedure e. b and c only

e. b and c only

Doppler analysis of intrarenal waveforms performed during renal sonography reveal a resistive index of 1.0. This finding is consistent with: a. normality b. chronic medical renal disease c. renal vein thrombosis d. renal obstruction e. B, C, and D

e. b, c, and d (RI is normally 0.7 or less)

What muscle is seen as thin, hypoechoic bands immediately anterior to each thyroid lobe? a. sternocleidomastoid b. longus coli c. sternohyoid d. sternothyroid e. both c and d

e. both c and d

You are performing an ultrasound study on a patient with a history of recent blunt left-sided trauma in a motor vehicle accident. Which finding below is NOT associated with this history? a. splenic laceration b. subcapsular hematoma of the spleen c. free fluid d. perisplenic hematoma e. calcified splenic cyst

e. calcified splenic cyst (calcified splenic cyst may be associated with a past history of trauma but is not an acute finding)

The arrow labeled D is pointing to what lobe of the liver? a. medial segment of left lobe b. lateral segment left lobe c. posterior segment right lobe d. anterior segment right lobe e. caudate lobe

e. caudate lobe

You are performing a Doppler study on a patient with median arcuate ligament syndrome. What vessel are you studying? a. inferior mesenteric artery b. renal artery c. portal vein d. superior mesenteric vein e. celiac trunk

e. celiac trunk (ask Liesel)

What vessels will you study in a patient who has been referred for a doppler study with symptoms of chronic mesenteric ischemia? a. portal, superior mesenteric, and splenic veins b. aorta, renal, common iliac arteries c. main renal and intrarenal vessels d. distal aorta, common iliac, external iliac, and common femoral arteries e. celiac trunk, superior mesenteric, and inferior mesenteric arteries

e. celiac trunk, superior mesenteric, and inferior mesenteric arteries

A patient is regerred for ultrasound with jaundice, pain, nausea and vomiting and a history of cholecystectomy. Which of the following findings is most likely? a. acute cholecystitis b. chronic cholecystitis c. adenomyomatosis d. emphysematous cholecystitis e. choledocholithiasis

e. choledocholithiasis

Your department protocol requires you to measure the main pancreatic duct whenever it is visible by ultrasound. What is the name of the duct you are measuring? a. duct of santorini b. cystic duct c. duct of oddi d. duct of vater e. duct of wirsung

e. duct of wirsung

Which letter represents the pancreatic tail? a. a b. b c. c d. d e. e

e. e

You are scanning a 44-year-old man with diabetes. He complains of severe epigastric pain radiating to the back, vomiting, chills, and fever. Ultrasound findings include a large gallbladder with nondependent hyperechoic foci associated with ringdown artifacts. These findings are most consistent with: a. adenomyomatosis complicated by gallstones b. gallbladder carcinoma c. acute cholecystitis complicated by gallbladder perforation d. acalculous cholecystitis e. emphysematous cholecystitis

e. emphysematous cholecystitis (form of acute cholecystitis with gas forming bacteria. Air is associated with ringdown artifact and is nondependent. More common in males and diabetics)

You are imaging the gut with a high-resolution transducer. How many layers do you expect to see in a good imaging of the gut? a. one b. two c. three d. four e. five

e. five (superficial mucosa, deep mucosa, submucosa, muscularis, serosa)

Which of the following course interlobar and intersegmental within the liver? a. bile ducts b. portal veins c. hepatic arteries d. lymphatics e. hepatic veins

e. hepatic veins

You are scanning a patient and notice that the right and left kidneys are attached at their lower poles. What anomaly is this? a. duplicated collecting system b. supernumerary kidney c. ureterocele d. pelvic kidney e. horseshoe kidney

e. horseshoe kidney

When imaging the pancreas, which vessels do you routinely visualize at the posterior border of the pancreatic head? a. abdominal aorta b. right renal artery c. superior mesenteric artery d. superior mesenteric vein e. inferior vena cava

e. inferior vena cava

Identify the structure labeled B: a. strap muscle b. longus coli muscle c. sternocleidomastoid muscle d. esophaguse. e. isthmuse.

e. isthmus

A patient is referred for an abdominal ultrasound. You notice a yellow discoloration of the eyes and skin. This condition is called: a. hypoalbuminemia b. biliary stasis c. erythema d. pruritus e. jaundice

e. jaundice

You are imaging the pancreas and detect a vessel posteriorly, coursing between the SMA and the aorta. What vessel are you imaging? a. IMA b. splenic vein c. SMV d. right hepatic artery e. left renal vein

e. left renal vein

Which of the following forms the caudal border of the left portal vein? a. ligamentum venosum b. hepatoduodenal ligament c. main lobar fissure d. coronary ligament e. ligamentum teres

e. ligamentum teres

Which of the following lab tests is NOT used in evaluation of liver function? a. Gama-glutamyl transpeptidase (GCT) b. aspartate aminotransferase (AST) c. Direct bilirubin d. indirect bilirubin e. lipase

e. lipase (lipase is used to evaluate acute pancreatitis)

During ultrasound evaluation of the liver, a bull's eye or target lesion is identifies in the anterior right lobe. The most likely etiology of this mass is: a. liver abscess b. hepatic adenoma c. focal nodular hyperplasia d. hepatocellular carcinoma e. liver metastasis from lung cancer

e. liver metastasis from lung cancer

You are performing a scrotal ultrasound exam on a 38-year-old male with acute scrotal pain to rule out orchitis. Which finding below would not be associated with orchitis? a. hypoechoic testis b. hyperemic testicular flow c. enlargement of testis d. thickened scrotal wall e. microlithiasis

e. microlithiasis

A patient has been referred for an ultrasound study to rule out the presence of a rectus sheath hematoma. Where will you look? a. lateral aspect of the affected thigh b. popliteal fossa c. calf d. neck e. mid abdominal wall

e. mid abdominal wall

What is the role of color and spectral Doppler in evaluating thyroid nodules? a. increased color Doppler flow within a nodule indicates malignancy b. identification of a vascular halo around a nodule indicates a benign condition c. hyperemia of a heterogeneous thyroid by color Doppler indicates malignancy. d. a, b, c e. neither color nor spectral Doppler have been shown to be sensitive in determining if a thyroid nodule is benign or malignant

e. neither color nor spectral Doppler have been show to besensitive in determining if a thyroid nodule is benign or malignant

What is the accepted treatment for a simple renal cyst? a. surgical removal b. aspiration c. fine needle biopsy d. core biopsy e. no further evaluation required

e. no further evaluation required

Which of the following vessels normally follows a retroaortic course? a. left renal vein b. right renal vein c. SMV d. IMA e. none of the above

e. none of the above

You have detected an intratesticular mass on a patient with mild scrotal swelling. Which of the following is an ultrasound feature of seminoma you should look for? a. hyperechoic intratesticular lesions b. ill defined tumor borders c. large hydrocele d. all of the above e. none of the above

e. none of the above ( seminoma is a hypoechoic mass with well defined margins)

During routine surveillance of the urinary bladder, you detect the presence of periodic ureteral "jets". This is a sign of: a. ureteral stone b. transitional cell carcinoma c. ureteral spasm d. ureteral compression e. normality

e. normality

An ultrasound evaluation of liver cirrhosis should include a search for which associated complication? a. biliary dilatation b. mesenteric ischemia c. splenic infarction d. kaposi's sarcoma e. portal hypertension

e. portal hypertension

This solitary bright echogenic mass was an incidental finding on a n ultrasound study of the gallbladder. Which of the following correctly describes its position within the liver? a. medial left lobe b. lateral left lobe c. caudate lobe d. anterior segment right lobe e. posterior segment right lobe

e. posterior segment right lobe

You are performing an ultrasound exam on a patient with a history of chronic pancreatitis and abdominal bruit. What vascular abnormality is associated with this history? a. abdominal aortic aneurysm b. renal artery stenosis c. IMA occlusion d. retroaortic renal vein e. pseudoaneurysm of the hepatic or splenic artery

e. pseudoaneurysm of the hepatic or splenic artery

You are performing an ultrasound exam of the liver on a small patient with a 5 MHz curved linear array. Although you have increased the overall gain to its maximum setting, the posterior border of the liver and diaphragm are not visualized. What should you do? a. call the service representative to repair your equipment b. decrease the transmit power on the ultrasound unit c. move the focal zone into the near field d. rescan the liver with a higher frequency transducer e. rescan the liver with a lower frequency transducer

e. rescan the liver with a lower frequency transducer

What is the vessel labeled E? a. right internal iliac artery b. left external iliac artery c. right common iliac vein d. left internal iliac veinretro e. right common iliac artery

e. right common iliac artery

You are imaging a patient with pneumobilia. What artifact is associated with this condition? a. refraction b. shadowing c. side lobes d. grating lobes e. ringdown

e. ringdown

Which of the following is NOT true regarding cavernous hemangiomas? a. small, well-defined, hyperechoic mass b. consist of a vascular network c. more common in women than men d. usually asymptomatic e. show prominent, high-velocity color doppler signals

e. show prominent, high-velocity color doppler signals (the flow in hemangiomas is very slow and may not be detected by color doppler imaging)

This patient was regerred for an abdominal ultrasound because of epigastric pain and tenderness. This image was obtained from the left upper quadrant. Which of the following describes the ultrasound findings? a. pleural effusion b. splenomegaly c. ascites d. splenic metastases e. splenic cysts

e. splenic cysts

Identify the structure labeled "D" in this image a. left renal vein b. right renal vein c. inferior mesenteric vein d. pancreatic duct e. splenic vein

e. splenic vein

You are having difficulty imaging the entire pancreas in a patient referred for abdominal ultrasound. Which part of the pancreas is least commonly visualized by ultrasound? a. head b. neck c. uncinate process d. body e. tail

e. tail

You are having difficulty imaging the entire pancreas in a patient referred for abdominal ultrasound. Which part of the pancreas is least commonly visualized by ultrasound? a. head b. neck c. uncinate process d. body e. tail

e. tail

Which statement below would help you in identification of the right renal vein? a. the right renal vein lies inferior and posterior to the renal artery b. the right renal vein course underneath the IVC c. the right renal vein courses anterior to the abdominal aorta d. the right renal vein divides into a circumaortic ring before draining into the IVC e. the right renal vein lies anterior to the renal artery

e. the right renal vein lies anterior to the renal artery

A patient has been referred for ultrasound evaluation to rule out acute appendicitis. What laboratory values are pathognomonic for acute appendicitis? a. leukocytosis b. increased alpha-fetoprotein c. increased alkaline phosphatase d. incrased serum bilirubin e. there are no pathognomonic laboratory values for acute appendicitis

e. there are no pathognomonic laboratory values for acute appendicitis

What vessel is seen to course throught the transverse foramina of the cervical spine? a. common carotid b. superior thyroid c. inferior thyroid d. thyrocervical e. vertebral

e. vertebral


Kaugnay na mga set ng pag-aaral

Principles in Community and Public Health Nursing, ATI Community Historical Influences on Community and Public Health, ATI Community Population, Public, and Global Health, Environmental Influences, RN Economic Influences

View Set

Eating Disorders: Identification, Treatment, Healing, & Recovery

View Set

Chapter 13 - Meiosis and Sexual Life Cycles

View Set

Business Law -- Chapter 9 -- Cyberlaw and Privacy-- Mind Tap

View Set

1 - Escoger Listen to each question and choose the most logical response.

View Set

Chapter 23- Worksheet - Administrative Law Agency Creation and Powers

View Set

Organizational Behavior Assignment 4

View Set

Ch. 14 Nursing Management During Labor & Birth NCLEX

View Set